В каком направлении ток течет: В каком направлении течет постоянный ток? — Хабр Q&A

Содержание

Направление тока в проводнике, как, откуда и куда течет электрический ток в проводниках.

Электрический ток представляет собой упорядоченное движение заряженных частиц. В твердых телах это движение электронов (отрицательно заряженных частиц) в жидких и газообразных телах это движение ионов (положительно заряженных частиц). Более того ток бывает постоянным и переменным, и у них совсем разное движение электрических зарядов. Чтобы хорошо понять и усвоить тему движение тока в проводниках пожалуй сначала нужно более подробно разобраться с основами электрофизики. Именно с этого я и начну.

Итак, как вообще происходит движение электрического тока? Известно, что вещества состоят из атомов. Это элементарные частицы вещества. Строение атома напоминает нашу солнечную систему, где в центре расположено ядро атома. Оно состоит из плотно прижатых друг к другу протонов (положительных электрических частиц) и нейтронов (электрически нейтральных частиц). Вокруг этого ядра с огромной скоростью по своим орбитам вращаются электроны (более мелкие частицы, имеющие отрицательный заряд). У разных веществ количество электронов и орбит, по которым они вращаются, может быть различным. Атомы твердых веществ имеют так называемую кристаллическую решетку. Это структура вещества, по которой в определенной порядке располагаются атомы относительно друг друга.

А где же тут может возникнуть электрический ток? Оказывается, что у некоторых веществ (проводников тока) электроны, что наиболее удалены от своего ядра, могут отрываться от атома и переходить на соседний атом. Это движение электронов называется свободным. Просто электроны перемещаются внутри вещества от одного атома к другому. Но вот если к этому веществу (электрическому проводнику) подключить внешнее электромагнитное поле, тем самым создав электрическую цепь, то все свободные электроны начнут двигаться в одном направлении. Именно это и есть движение электрического тока внутри проводника.

Теперь давайте разберемся с тем, что собой представляет постоянный и переменный ток. Итак, постоянный ток всегда движется только в одном направлении. Как говорилось в самом начале — в твердых телах движутся электроны, а в жидких и газообразных движутся ионы. Электроны, это отрицательно  заряженные частицы. Следовательно, в твердых телах электрический ток течет от минуса к плюсу источника питания (перемещаются электроны по электрической цепи). В жидкостях и газах ток движется сразу в двух направлениях, а точнее, одновременно, электроны текут к плюсу, а ионы (отдельные атомы, что не связаны между собой кристаллической решеткой, они каждый сам по себе) текут к минусу источника питания.

Учеными же было принято официально считать, что движение происходит от плюса к минусу (наоборот, чем это происходит в действительности). Так что, с научной точки зрения правильно говорить, что электрический ток движется от плюса к минусу, а с  реальной точки зрения (электрофизическая природа) правильнее полагать, что ток течет от минуса к плюсу (в твердых телах). Наверное это сделано для какого-то удобства.

Теперь, что касается переменного электрического тока. Тут уже немного все сложнее. Если в случае постоянного тока движение заряженных частиц имеет только одно направление (физически электроны со знаком минус текут к плюсу), то при переменном токе направление движения периодически меняется на противоположное. Вы наверное слышали, что в обычной городской электросети переменное напряжение величиной 220 вольт и стандартной частотой 50 герц. Так вот эти 50 герц говорят о том, что электрический ток за одну секунду успевает 50 раз пройти полный цикл, имеющий синусоидальную форму. Фактически за одну секунду направление тока меняется аж 100 раз (за один цикл меняется два раза).

P.S. Направление тока в электрических схемах имеет важное значение. Во многих случаях если схема рассчитана на одно направление тока, а вы случайно его поменяете на противоположный или вместо постоянного тока подключите переменный, то скорее всего устройство просто выйдет из строя. Многие полупроводники, что работают в схемах, при обратном направлении тока могут пробиваться и сгорать. Так что при подключении электрического питания направление тока должно быть вами строго соблюдаться.

В каком направлении течет электрический ток?

Можно ли ответить на этот вопрос? — В отдаленные времена, когда физики изучали сравнительно очень узкий круг известных им электрических явлений, были введены понятия положительного и отрицательного электричества.

Знак плюс присвоили «стеклянному» электричеству — тому электрическому заряду, который возникает на стекле в результате натирания его шелком.

Отрицательным электричеством стали считать «сургучное»— заряд, возникающий на сургуче, натертом шерстью. В дальнейшем условились считать, что электрический ток течет от плюса к минусу.

Такая терминология оказалась удобной. Она устраивала и физиков и техников и сохранилась до наших дней. На ее базе сформулированы все основные законы, правила и зависимости учения об электричестве.

Однако несоответствие подобной терминологии физической сущности электрических явлений стало очевидным уже в последние годы прошлого столетия, когда были открыты электроны.

Это открытие доказало, что электрический ток имеет «зернистую» структуру и представляет собой поток мельчайших отрицательных зарядов — электронов. Электроны движутся от минуса к плюсу, т. е. в направлении, обратном тому, какое было установлено на заре электротехники.

Это породило двойственность и путаницу. Во многих случаях, когда речь шла о направлении тока, приходилось специально оговаривать, как понимать направление: «по току» или «по электронам».

Особенно болезненно эта терминологическая двойственность чувствуется в радиотехнике, где для уяснения работы схем и приборов часто бывает необходимо

учитывать именно направление движения электронов. Например, в какую сторону «проводит» электронная лампа? Если считать «по току», то лампа проводит от анода к катоду, а если «по электронам», то от катода к аноду.

Часто высказывается мысль о необходимости устранить двойственность терминологии и установить единообразие в представлении о направлении тока.

Можно ли осуществить подобное единообразие? Это сделать не так легко, как кажется. Конечно, нетрудно изъять из всей выходящей литературы упоминание об электрическом токе в его старом толковании и ввести…

А что же ввести? Направление движения электронов? А почему именно электронов? Мы теперь знаем, что электрический ток есть движение электрических зарядов, к которым относятся и электроны, и протоны, и ионы, и «дырки» Электроны и отрицательные ионы движутся от нашего условного минуса к столь, же условному плюсу, а положительные ионы, протоны и «дырки» движутся в обратном направлении.

Можно составить цепь из металлических проводников, гальванических элементов, полупроводниковых выпрямителей и т. п., в отдельных участках которой электрические заряды, образующие электрический ток, будут двигаться в противоположных направлениях.

Что принять за направление тока в полупроводниковом диоде, в котором электроны движутся в одном направлений, а «дырки» — в обратном? Как видим, вопрос о направлении тока не так-то прост.

Источник: Бурлянд В.А., Жеребцов И.П. Хрестоматия радиолюбителя. 1963 г.

В каком направлении течет ток

Тема: в какую сторону идёт ток в проводах, электрических цепях, схемах.

Электрический ток представляет собой упорядоченное движение заряженных частиц. В твердых телах это движение электронов (отрицательно заряженных частиц) в жидких и газообразных телах это движение ионов (положительно заряженных частиц). Более того ток бывает постоянным и переменным, и у них совсем разное движение электрических зарядов. Чтобы хорошо понять и усвоить тему движение тока в проводниках пожалуй сначала нужно более подробно разобраться с основами электрофизики. Именно с этого я и начну.

Итак, как вообще происходит движение электрического тока? Известно, что вещества состоят из атомов. Это элементарные частицы вещества. Строение атома напоминает нашу солнечную систему, где в центре расположено ядро атома. Оно состоит из плотно прижатых друг к другу протонов (положительных электрических частиц) и нейтронов (электрически нейтральных частиц). Вокруг этого ядра с огромной скоростью по своим орбитам вращаются электроны (более мелкие частицы, имеющие отрицательный заряд). У разных веществ количество электронов и орбит, по которым они вращаются, может быть различным. Атомы твердых веществ имеют так называемую кристаллическую решетку. Это структура вещества, по которой в определенной порядке располагаются атомы относительно друг друга.

А где же тут может возникнуть электрический ток? Оказывается, что у некоторых веществ (проводников тока) электроны, что наиболее удалены от своего ядра, могут отрываться от атома и переходить на соседний атом. Это движение электронов называется свободным. Просто электроны перемещаются внутри вещества от одного атома к другому. Но вот если к этому веществу (электрическому проводнику) подключить внешнее электромагнитное поле, тем самым создав электрическую цепь, то все свободные электроны начнут двигаться в одном направлении. Именно это и есть движение электрического тока внутри проводника.

Теперь давайте разберемся с тем, что собой представляет постоянный и переменный ток. Итак, постоянный ток всегда движется только в одном направлении. Как говорилось в самом начале — в твердых телах движутся электроны, а в жидких и газообразных движутся ионы. Электроны, это отрицательно заряженные частицы. Следовательно, в твердых телах электрический ток течет от минуса к плюсу источника питания (перемещаются электроны по электрической цепи). В жидкостях и газах ток движется сразу в двух направлениях, а точнее, одновременно, электроны текут к плюсу, а ионы (отдельные атомы, что не связаны между собой кристаллической решеткой, они каждый сам по себе) текут к минусу источника питания.

Учеными же было принято официально считать, что движение происходит от плюса к минусу (наоборот, чем это происходит в действительности). Так что, с научной точки зрения правильно говорить, что электрический ток движется от плюса к минусу, а с реальной точки зрения (электрофизическая природа) правильнее полагать, что ток течет от минуса к плюсу (в твердых телах). Наверное это сделано для какого-то удобства.

Теперь, что касается переменного электрического тока. Тут уже немного все сложнее. Если в случае постоянного тока движение заряженных частиц имеет только одно направление (физически электроны со знаком минус текут к плюсу), то при переменном токе направление движения периодически меняется на противоположное. Вы наверное слышали, что в обычной городской электросети переменное напряжение величиной 220 вольт и стандартной частотой 50 герц. Так вот эти 50 герц говорят о том, что электрический ток за одну секунду успевает 50 раз пройти полный цикл, имеющий синусоидальную форму. Фактически за одну секунду направление тока меняется аж 100 раз (за один цикл меняется два раза).

В электрической цепи, включающей источник тока и потребитель электроэнергии, возникает электрический ток. Но в каком направлении возникает этот самый ток? Традиционно считается, что во внешней цепи ток имеет направление от плюса источника к минусу в то время, как внутри источника питания — от минуса к плюсу.

И действительно, электрический ток — это упорядоченное движение электрически заряженных частиц. В случае, если проводник изготовлен из металла, такими частицами служат электроны — отрицательно заряженные частицы. Однако во внешней цепи электроны движутся именно от минуса (отрицательного полюса) к плюсу (положительному полюсу), а не от плюса к минусу.

Если включить во внешнюю цепь полупроводниковый диод, то станет ясным, что ток возможен лишь тогда, когда диод подключен катодом в сторону минуса. Из этого следует, что за направление электрического тока в цепи принимают направление противоположное реальному движению электронов.

Если проследить историю становления электротехники как самостоятельной науки, можно понять, откуда возник такой парадоксальный подход.

Американский исследователь Бенжамин Франклин выдвинул в свое время унитарную (единую) теорию электричества. По этой теории электрическая материя является невесомой жидкостью, которая может вытекать из одних тел, при этом накапливаться в других.

По Франклину, электрическая жидкость есть во всех телах, но наэлектризованными тела становится лишь тогда, когда в них имеет место избыток или недостаток электрической жидкости (электрического флюида). Недостаток электрического флюида (по Франклину) означал отрицательную электризацию, а избыток – положительную.

Так было положено начало понятиям положительного заряда и отрицательного заряда. В момент соединения тел заряженных положительно с телами, заряженными отрицательно, электрическая жидкость перетекает от тела с большим количеством электрической жидкости к телам с пониженным ее количеством. Это похоже на систему сообщающихся сосудов. В науку вошло устойчивое понятие электрического тока, движения электрических зарядов.

Эта гипотеза Франклина предварила электронную теорию проводимости, однако она оказалась совсем не безупречной. Французский физик Шарль Дюфе обнаружил, что в реальности есть два вида электричества, которые в отдельности подчиняется теории Франклина, однако при соприкосновении взаимно нейтрализуются. Появилась новая дуалистическая (двойственная) теория электричества, выдвинутая естествоиспытателем Робертом Симмером на основании опытов Шарля Дюфе.

При натирании, с целью электризации, электризуемых тел, заряженным становится не только натираемое тело, но и натирающее. Дуалистическая теория утверждала, что в обычном состоянии в телах содержатся два рода электрического флюида и в разных количествах, которые нейтрализуют друг друга. Объяснялась электризация изменением соотношения отрицательных и положительных электричеств в электризуемых телах.

Как гипотеза Франклина, так и гипотеза Симмера успешно объясняли электростатические явления и даже конкурировали между собой.

Изобретенный в 1799 году вольтов столб и открытие явления электролиза привели к выводам о том, что при электролизе растворов и жидкостей в них наблюдается два противоположных по направлению движения зарядов – отрицательное и положительное. Это было торжество дуалистической теории, ведь при разложении воды теперь можно было наблюдать, как на положительном электроде происходит выделение пузырьков кислорода, в то же время на отрицательном – водорода.

Но здесь не все было гладко. Количество выделяемых газов получалось разным. Водорода выделялось вдвое больше, чем кислорода. Это ставило физиков в тупик. Тогда химики еще не имели представления о том, что в молекуле воды присутствуют два атома водорода и всего один атом кислорода.

Эти теории не были понятны всем.

Но в 1820 году Андре-Мари Ампер в работе, представленной членам Парижской академии наук, сперва решает выбрать одно из направлений токов в качестве основного, но затем дает правило, согласно которому можно точно определить воздействие магнитов на электрические токи.

Чтобы все время не говорить о двух противоположных по направлению токах обоих электричеств, во избежание лишних повторений, Ампер решил за направление электрического тока строго принять направление движения именно положительного электричества. Так, впервые Ампером было введено до сих пор общепринятое правило направления электрического тока.

Этого положения придерживался позже и сам Максвелл, придумавший правило «буравчика», определяющее направление магнитного поля катушки. Но вопрос об истинном направлении электрического тока так и оставался открытым. Фарадей писал, что такое положение вещей лишь условно, оно удобно ученым, и помогает им ясно определять направления токов. Но это лишь удобное средство.

После открытия Фарадеем электромагнитной индукции, появилась необходимость определять направление индуцированного тока. Русский физик Ленц дал правило: если металлический проводник движется вблизи тока или магнита, то в нем возникает гальванический ток. И направление возникающего тока таково, что неподвижный провод пришел бы от его действия в движение, противоположное исходному перемещению. Просто, облегчающее понимание правило.

Даже после открытия электрона, эта условность существует более полутора столетий. С изобретением такого устройства, как электронная лампа, с широким внедрением полупроводников, стали возникать трудности. Но электротехника, как и прежде, оперирует старыми определениями. Порой это вызывает настоящую путаницу. Но внесение коррективов вызовет больше неудобств.

Электрический ток в разных веществах

Электрический ток возникает в самых разных веществах, которые могут находиться в различных агрегатных состояниях. Рассмотрим некоторые примеры, демонстрирующие возникновение направленного потока заряженных частиц в твердых, жидких и газообразных средах:

  • В металлах имеется много свободных электронов, которые являются главным источником тока;
  • Электролиты — это жидкости, проводящие электрический ток. Водные растворы кислот, щелочей, солей — все это примеры электролитов. Попадая в воду молекулы этих веществ распадаются на ионы, представляющие собой заряженные атомы или группы атомов, имеющие положительный (катионы) или отрицательный (анионы) электрические заряды. Катионы и анионы образуют электрический ток в электролитах;
  • В газах и плазме ток создается за счет движения электронов и положительно заряженных ионов;
  • В вакууме — за счет электронов, вылетающих с поверхности металлических электродов.

Рис. 1. Примеры электрического тока в разных веществах (металлах, электролитах, газах, плазме, вакууме).

В приведенных примерах токи возникают в результате движения заряженных частиц относительно той или иной среды (внутри тел). Такой ток называется током проводимости. Движение макроскопических заряженных тел называется конвекционным током. Примером конвекционного тока могут служить капли дождя во время разряда молнии.

В каком направлении течет ток

За направление тока принято направление движения положительно заряженных частиц; если же ток создается отрицательно заряженными частицами (например, электронами), то направление тока считается противоположным направлению движения частиц.

Рис. 2. Направление движения тока для любой электрической цепи.

Возникает вопрос: почему не был принят очевидный вариант направления, совпадающий с направлением движения электронов? Для того, чтобы это стало понятно, надо немного окунуться в историю физики.

Почему надо знать историю физических открытий

Природу электрических явлений пытались объяснить многие исследователи задолго до открытия электрона (1897 г.). Впервые к пониманию о существовании двух типов зарядов — положительных и отрицательных пришел американский физик Бенджамин Франклин в 1747 г. На основе своих наблюдений он предположил (выдвинул гипотезу), что существует некая “электрическая материя”, состоящая из мелких, невидимых частиц. Он же первым ввел обозначение для электрических зарядов “−” и “+”. Франклин предложил считать, что если тело наполняется электрической материей, то оно заряжается положительно, а если оно теряет электричество, то заряжается отрицательно. В случае замыкания (соединения) цепи положительный заряд потечет туда, где его нет, то есть к “минусу”. Эта плодотворная гипотеза стала популярной, получила свое признание среди ученых, вошла в справочники и учебные пособия.

Конечно, после открытия отрицательно заряженного электрона, эта “нестыковка” реального направления движения с ранее общепринятым была обнаружена. Однако, мировым научным сообществом было принято решение оставить в силе предыдущую формулировку о направлении тока, поскольку в большинстве практических случаев это ни на что не влияет.

В случае необходимости, для объяснения отдельных физических эффектов в полупроводниках и искусственных материалах (гетероструктурах), принимается во внимание настоящее направление движения электронов.

Бенджамин Франклин знаменит еще как выдающийся политический деятель, дипломат и писатель. Он является одним из авторов конституции США. В знак признания заслуг Франклина на купюре номиналом в 100 долларов с 1914 г. изображен его портрет.

Рис. 3. Изображение купюры 100 долларов США с портретом Бенджамина Франклина.

Что мы узнали?

Итак, мы узнали, что направление тока в электрической цепи соответствует направлению движения положительных зарядов, то есть от плюсового потенциала (плюса) к минусовому потенциалу (минусу). Несмотря на то, что чаще всего электрический ток создается отрицательно заряженными электронами, выбор направления тока было решено оставить именно таким. Так сложилось исторически.

Как определить направление электрического тока

Всем известно, что суть электрического тока заключается в упорядоченном движении заряженных частиц в каких-либо проводниках. Чаще всего для этих целей используются различные металлы, где в качестве тока выступают отрицательно заряженные частицы – электроны. В кислотных, щелочных и солевых растворах электрический ток возникает в результате движения положительных и отрицательных ионов.

С самого начала, когда это явление было открыто, у многих ученых возникал вопрос: какие движущиеся заряженные частицы образуют направление тока? Чтобы до конца разобраться в данной проблеме, следует остановиться на источниках тока, поскольку именно они инициируют движение заряженных частиц в проводниках.

Откуда берется электрический ток

Движение заряженных частиц появляется в результате действия, производимого аккумуляторами, батареями, генераторами и другими устройствами, преобразующими различные виды энергии в электрическую. Закон сохранения энергии наглядно действует в процессе таких преобразований.

Сами частицы начинают двигаться, когда цепь становится замкнутой, а в проводнике возникает электрическое поле, оказывающее определенное воздействие на свободные электроны. В связи с этим было установлено, что все источники тока обладают установленной электродвижущей силой или ЭДС.

Электроны не появляются из источников тока, они присутствуют в самих проводниках и, являясь свободными, начинают двигаться под действием созданного поля. В качестве наиболее яркого сравнительного примера выступает насос перекачивающий жидкость в трубах, замкнутых между собой. В зависимости от диаметра труб и количества разветвлений, жидкость может двигаться по ним с большей или меньшей скоростью. Эти свойства в полной мере характеризуют течение тока, которое изменяется в соответствии с сечением проводника.

На практике это выглядит следующим образом. Провод, сечением 1,5 мм2, рассчитан на максимальную силу тока в 16 А. К нему может быть подключена нагрузка не более 3-3,5 кВт. При подключении более мощного оборудования проводник не выдержит и выйдет из строя.

Разобравшись с источниками тока, необходимо определить его направление, которое приняли ученые после проведенных исследований в этой области. Условно было принято направление движения положительных зарядов, поскольку ток от положительного полюса движется к отрицательному полюсу источника тока.

Движение частиц и направление тока

Прежде всего, следует отметить, что не все движущиеся заряженные частицы вызывают образование тока. Например, под действием тепла заряды будут двигаться, но это движение – хаотическое и ненаправленное. Если же к тепловому движению добавляется действие электрическое поле, то под его влиянием хаотические перемещения частиц примут определенную направленность.

Заряженные частицы, образующие ток, движутся в направлении, в зависимости от знака их заряда. То есть, движение положительно заряженных частиц происходит от «+» к «-», а отрицательно заряженных, наоборот, от «-» к «+». Встречное движение характерно для газовой и электролитической среды, поэтому часто возникает вопрос, каким будет настоящее направление тока?

По общему соглашению было принято решение считать направление движения частиц с положительными зарядами, за направление электрического тока. В этом случае возникает некоторое противоречие, затрагивающее металлические проводники, в которых перенос зарядов осуществляется свободными электронами. Хорошо известно, что они двигаются от минуса к плюсу. Тем не менее, приходится считать направление тока в этом случае, противоположным движению свободных электронов. Однако, несмотря на некоторые неудобства, данное правило четко определяет, в каком направлении движется электрический ток.

Всем известно, что суть электрического тока заключается в упорядоченном движении заряженных частиц в каких-либо проводниках. Чаще всего для этих целей используются различные металлы, где в качестве тока выступают отрицательно заряженные частицы – электроны. В кислотных, щелочных и солевых растворах электрический ток возникает в результате движения положительных и отрицательных ионов.

С самого начала, когда это явление было открыто, у многих ученых возникал вопрос: какие движущиеся заряженные частицы образуют направление тока? Чтобы до конца разобраться в данной проблеме, следует остановиться на источниках тока, поскольку именно они инициируют движение заряженных частиц в проводниках.

Откуда берется электрический ток

Движение заряженных частиц появляется в результате действия, производимого аккумуляторами, батареями, генераторами и другими устройствами, преобразующими различные виды энергии в электрическую. Закон сохранения энергии наглядно действует в процессе таких преобразований.

Сами частицы начинают двигаться, когда цепь становится замкнутой, а в проводнике возникает электрическое поле, оказывающее определенное воздействие на свободные электроны. В связи с этим было установлено, что все источники тока обладают установленной электродвижущей силой или ЭДС.

Электроны не появляются из источников тока, они присутствуют в самих проводниках и, являясь свободными, начинают двигаться под действием созданного поля. В качестве наиболее яркого сравнительного примера выступает насос перекачивающий жидкость в трубах, замкнутых между собой. В зависимости от диаметра труб и количества разветвлений, жидкость может двигаться по ним с большей или меньшей скоростью. Эти свойства в полной мере характеризуют течение тока, которое изменяется в соответствии с сечением проводника.

На практике это выглядит следующим образом. Провод, сечением 1,5 мм2, рассчитан на максимальную силу тока в 16 А. К нему может быть подключена нагрузка не более 3-3,5 кВт. При подключении более мощного оборудования проводник не выдержит и выйдет из строя.

Разобравшись с источниками тока, необходимо определить его направление, которое приняли ученые после проведенных исследований в этой области. Условно было принято направление движения положительных зарядов, поскольку ток от положительного полюса движется к отрицательному полюсу источника тока.

Движение частиц и направление тока

Прежде всего, следует отметить, что не все движущиеся заряженные частицы вызывают образование тока. Например, под действием тепла заряды будут двигаться, но это движение – хаотическое и ненаправленное. Если же к тепловому движению добавляется действие электрическое поле, то под его влиянием хаотические перемещения частиц примут определенную направленность.

Заряженные частицы, образующие ток, движутся в направлении, в зависимости от знака их заряда. То есть, движение положительно заряженных частиц происходит от «+» к «-», а отрицательно заряженных, наоборот, от «-» к «+». Встречное движение характерно для газовой и электролитической среды, поэтому часто возникает вопрос, каким будет настоящее направление тока?

По общему соглашению было принято решение считать направление движения частиц с положительными зарядами, за направление электрического тока. В этом случае возникает некоторое противоречие, затрагивающее металлические проводники, в которых перенос зарядов осуществляется свободными электронами. Хорошо известно, что они двигаются от минуса к плюсу. Тем не менее, приходится считать направление тока в этом случае, противоположным движению свободных электронов. Однако, несмотря на некоторые неудобства, данное правило четко определяет, в каком направлении движется электрический ток.

Автор статьи — профессиональный репетитор, автор учебных пособий для подготовки к ЕГЭ Игорь Вячеславович Яковлев

Темы кодификатора ЕГЭ: постоянный электрический ток, сила тока, напряжение.

Электрический ток обеспечивает комфортом жизнь современного человека. Технологические достижения цивилизации — энергетика, транспорт, радио, телевидение, компьютеры, мобильная связь — основаны на использовании электрического тока.

Электрический ток — это направленное движение заряженных частиц, при котором происходит перенос заряда из одних областей пространства в другие.

Электрический ток может возникать в самых различных средах: твёрдых телах, жидкостях, газах. Порой и среды никакой не нужно — ток может существовать даже в вакууме! Мы поговорим об этом в своё время, а пока приведём лишь некоторые примеры.

• Замкнём полюса батарейки металлическим проводом. Свободные электроны провода начнут направленное движение от «минуса» батарейки к «плюсу».
Это — пример тока в металлах.

• Бросим в стакан воды щепотку поваренной соли . Молекулы соли диссоциируют на ионы, так что в растворе появятся свободные заряды: положительные ионы и отрицательные ионы . Теперь засунем в воду два электрода, соединённые с полюсами батарейки. Ионы начнут направленное движение к отрицательному электроду, а ионы — к положительному.
Это — пример прохождения тока через раствор электролита.

• Грозовые тучи создают столь мощные электрические поля, что оказывается возможным пробой воздушного промежутка длиной в несколько километров. В результате сквозь воздух проходит гигантский разряд — молния.
Это — пример электрического тока в газе.

Во всех трёх рассмотренных примерах электрический ток обусловлен движением заряженных частиц внутри тела и называется током проводимости.

• Вот несколько иной пример. Будем перемещать в пространстве заряженное тело. Такая ситуация согласуется с определением тока! Направленное движение зарядов — есть, перенос заряда в пространстве — присутствует. Ток, созданный движением макроскопического заряженного тела, называется конвекционным.

Заметим, что не всякое движение заряженных частиц образует ток. Например, хаотическое тепловое движение зарядов проводника — не направленное (оно совершается в каких угодно направлениях), и потому током не является (при возникновении тока свободные заряды продолжают совершать тепловое движение! Просто в этом случае к хаотическим перемещениям заряженных частиц добавляется их упорядоченный дрейф в определённом
направлении).
Не будет током и поступательное движение электрически нейтрального тела: хотя заряженные частицы в его атомах и совершают направленное движение, не происходит переноса заряда из одних участков пространства в другие.

Направление электрического тока

Направление движения заряженных частиц, образующих ток, зависит от знака их заряда. Положительно заряженные частицы будут двигаться от «плюса» к «минусу», а отрицательно заряженные — наоборот, от «минуса» к «плюсу». В электролитах и газах, например, присутствуют как положительные, так и отрицательные свободные заряды, и ток создаётся их встречным движением в обоих направлениях. Какое же из этих направлений принять за направление электрического тока?

Направлением тока принято считать направление движения положительных зарядов.

Попросту говоря, по соглашению ток течёт от «плюса» к «минусу» (рис. 1 ; положительная клемма источника тока изображена длинной чертой, отрицательная клемма — короткой).

Рис. 1. Направление тока

Данное соглашение вступает в некоторое противоречие с наиболее распространённым случаем металлических проводников. В металле носителями заряда являются свободные электроны, и двигаются они от «минуса» к «плюсу». Но в соответствии с соглашением мы вынуждены считать, что направление тока в металлическом проводнике противоположно движению свободных электронов. Это, конечно, не очень удобно.

Тут, однако, ничего не поделаешь — придётся принять эту ситуацию как данность. Так уж исторически сложилось. Выбор направления тока был предложен Ампером (договорённость о направлении тока понадобилась Амперу для того, чтобы дать чёткое правило определения направления силы, действующей на проводник с током в магнитном поле. Сегодня эту силу мы называем силой Ампера, направление которой определяется по правилу левой руки) в первой половине XIX века, за 70 лет до открытия электрона. К этому выбору все привыкли, и когда в 1916 году выяснилось, что ток в металлах вызван движением свободных электронов, ничего менять уже не стали.

Действия электрического тока

Как мы можем определить, протекает электрический ток или нет? О возникновении электрического тока можно судить по следующим его проявлениям.

1. Тепловое действие тока. Электрический ток вызывает нагревание вещества, в котором он протекает. Именно так нагреваются спирали нагревательных приборов и ламп накаливания. Именно поэтому мы видим молнию. В основе действия тепловых амперметров лежит тепловое расширение проводника с током, приводящее к перемещению стрелки прибора.

2. Магнитное действие тока. Электрический ток создаёт магнитное поле: стрелка компаса, расположенная рядом с проводом, при включении тока поворачивается перпендикулярно проводу. Магнитное поле тока можно многократно усилить, если обмотать провод вокруг железного стержня — получится электромагнит. На этом принципе основано действие амперметров магнитоэлектрической системы: электромагнит поворачивается в поле постоянного магнита, в результате чего стрелка прибора перемещается по шкале.

3. Химическое действие тока. При прохождении тока через электролиты можно наблюдать изменение химического состава вещества. Так, в растворе положительные ионы двигаются к отрицательному электроду, и этот электрод покрывается медью.

Электрический ток называется постоянным, если за равные промежутки времени через поперечное сечение проводника проходит одинаковый заряд.

Постоянный ток наиболее прост для изучения. С него мы и начинаем.

Сила и плотность тока

Количественной характеристикой электрического тока является сила тока. В случае постоянного тока абсолютная величина силы тока есть отношение абсолютной величины заряда , прошедшего через поперечное сечение проводника за время , к этому самому времени:

Измеряется сила тока в амперах (A). При силе тока в А через поперечное сечение проводника за с проходит заряд в Кл.

Подчеркнём, что формула (1) определяет абсолютную величину, или модуль силы тока.
Сила тока может иметь ещё и знак! Этот знак не связан со знаком зарядов, образующих ток, и выбирается из иных соображений. А именно, в ряде ситуаций (например, если заранее не ясно, куда потечёт ток) удобно зафиксировать некоторое направление обхода цепи (скажем, против часовой стрелки) и считать силу тока положительной, если направление тока совпадает с направлением обхода, и отрицательной, если ток течёт против направления обхода (сравните с тригонометрическим кругом: углы считаются положительными, если отсчитываются против часовой стрелки, и отрицательными, если по часовой стрелке).

В случае постоянного тока сила тока есть величина постоянная. Она показывает, какой заряд проходит через поперечное сечение проводника за с.

Часто бывает удобно не связываться с площадью поперечного сечения и ввести величину плотности тока:

где — сила тока, — площадь поперечного сечения проводника (разумеется, это сечение перпендикулярно направлению тока). С учётом формулы (1) имеем также:

Плотность тока показывает, какой заряд проходит за единицу времени через единицу площади поперечного сечения проводника. Согласно формуле (2) , плотность тока измеряется в А/м2.

Скорость направленного движения зарядов

Когда мы включаем в комнате свет, нам кажется, что лампочка загорается мгновенно. Скорость распространения тока по проводам очень велика: она близка к км/с (скорости света в вакууме). Если бы лампочка находилась на Луне, она зажглась бы через секунду с небольшим.

Однако не следует думать, что с такой грандиозной скоростью двигаются свободные заряды, образующие ток. Оказывается, их скорость составляет всего-навсего доли миллиметра в секунду.

Почему же ток распространяется по проводам так быстро? Дело в том, что свободные заряды взаимодействуют друг с другом и, находясь под действием электрического поля источника тока, при замыкании цепи приходят в движение почти одновременно вдоль всего проводника. Скорость распространения тока есть скорость передачи электрического взаимодействия между свободными зарядами, и она близка к скорости света в вакууме. Скорость же, с которой сами заряды перемещаются внутри проводника, может быть на много порядков меньше.

Итак, подчеркнём ещё раз, что мы различаем две скорости.

1. Скорость распространения тока. Это — скорость передачи электрического сигнала по цепи. Близка к км/с.

2. Скорость направленного движения свободных зарядов. Это — средняя скорость перемещения зарядов, образующих ток. Называется ещё скоростью дрейфа.

Мы сейчас выведем формулу, выражающую силу тока через скорость направленного движения зарядов проводника.

Пусть проводник имеет площадь поперечного сечения (рис. 2). Свободные заряды проводника будем считать положительными; величину свободного заряда обозначим (в наиболее важном для практики случая металлического проводника это есть заряд электрона). Концентрация свободных зарядов (т. е. их число в единице объёма) равна .

Рис. 2. К выводу формулы

Какой заряд пройдёт через поперечное сечение нашего проводника за время ?

С одной стороны, разумеется,

С другой стороны, сечение пересекут все те свободные заряды, которые спустя время окажутся внутри цилиндра с высотой . Их число равно:

Следовательно, их общий заряд будет равен:

Приравнивая правые части формул (3) и (4) и сокращая на , получим:

Соответственно, плотность тока оказывается равна:

Давайте в качестве примера посчитаем, какова скорость движения свободных электронов в медном проводе при силе тока A.

Заряд электрона известен: Кл.

Чему равна концентрация свободных электронов? Она совпадает с концентрацией атомов меди, поскольку от каждого атома отщепляется по одному валентному электрону. Ну а концентрацию атомов мы находить умеем:

Положим мм . Из формулы (5) получим:

Это порядка одной десятой миллиметра в секунду.

Стационарное электрическое поле

Мы всё время говорим о направленном движении зарядов, но ещё не касались вопроса о том, почему свободные заряды совершают такое движение. Почему, собственно, возникает электрический ток?

Для упорядоченного перемещения зарядов внутри проводника необходима сила, действующая на заряды в определённом направлении. Откуда берётся эта сила? Со стороны электрического поля!

Чтобы в проводнике протекал постоянный ток, внутри проводника должно существовать стационарное (то есть — постоянное, не зависящее от времени) электрическое поле. Иными словами, между концами проводника нужно поддерживать постоянную разность потенциалов.

Стационарное электрическое поле должно создаваться зарядами проводников, входящих в электрическую цепь. Однако заряженные проводники сами по себе не смогут обеспечить протекание постоянного тока.

Рассмотрим, к примеру, два проводящих шара, заряженных разноимённо. Соединим их проводом. Между концами провода возникнет разность потенциалов, а внутри провода — электрическое поле. По проводу потечёт ток. Но по мере прохождения тока разность потенциалов между шарами будет уменьшаться, вслед за ней станет убывать и напряжённость поля в проводе. В конце концов потенциалы шаров станут равны друг другу, поле в проводе обратится в нуль, и ток исчезнет. Мы оказались в электростатике: шары плюс провод образуют единый проводник, в каждой точке которого потенциал принимает одно и то же значение; напряжённость
поля внутри проводника равна нулю, никакого тока нет.

То, что электростатическое поле само по себе не годится на роль стационарного поля, создающего ток, ясно и из более общих соображений. Ведь электростатическое поле потенциально, его работа при перемещении заряда по замкнутому пути равна нулю. Следовательно, оно не может вызывать циркулирование зарядов по замкнутой электрической цепи — для этого требуется совершать ненулевую работу.

Кто же будет совершать эту ненулевую работу? Кто будет поддерживать в цепи разность потенциалов и обеспечивать стационарное электрическое поле, создающее ток в проводниках?

Ответ — источник тока, важнейший элемент электрической цепи.

Чтобы в проводнике протекал постоянный ток, концы проводника должны быть присоединены к клеммам источника тока (батарейки, аккумулятора и т. д.).

Клеммы источника — это заряженные проводники. Если цепь замкнута, то заряды с клемм перемещаются по цепи — как в рассмотренном выше примере с шарами. Но теперь разность потенциалов между клеммами не уменьшается: источник тока непрерывно восполняет заряды на клеммах, поддерживая разность потенциалов между концами цепи на неизменном уровне.

В этом и состоит предназначение источника постоянного тока. Внутри него протекают процессы неэлектрического (чаще всего — химического) происхождения, которые обеспечивают непрерывное разделение зарядов. Эти заряды поставляются на клеммы источника в необходимом количестве.

Количественную характеристику неэлектрических процессов разделения зарядов внутри источника — так называемую ЭДС — мы изучим позже, в соответствующем листке.

А сейчас вернёмся к стационарному электрическому полю. Каким же образом оно возникает в проводниках цепи при наличии источника тока?

Заряженные клеммы источника создают на концах проводника электрическое поле. Свободные заряды проводника, находящиеся вблизи клемм, приходят в движение и действуют своим электрическим полем на соседние заряды. Со скоростью, близкой к скорости света, это взаимодействие передаётся вдоль всей цепи, и в цепи устанавливается постоянный электрический ток. Стабилизируется и электрическое поле, создаваемое движущимися зарядами.

Стационарное электрическое поле — это поле свободных зарядов проводника, совершающих направленное движение.

Стационарное электрическое поле не меняется со временем потому, что при постоянном токе не меняется картина распределения зарядов в проводнике: на место заряда, покинувшего данный участок проводника, в следующий момент времени поступает точно такой же заряд. По этой причине стационарное поле во многом (но не во всём) аналогично полю электростатическому.

А именно, справедливы следующие два утверждения, которые понадобятся нам в дальнейшем (их доказательство даётся в вузовском курсе физики).

1. Как и электростатическое поле, стационарное электрическое поле потенциально. Это позволяет говорить о разности потенциалов (т. е. напряжении) на любом участке цепи (именно эту разность потенциалов мы измеряем вольтметром).
Потенциальность, напомним, означает, что работа стационарного поля по перемещению заряда не зависит от формы траектории. Именно поэтому при параллельном соединении проводников напряжение на каждом из них одинаково: оно равно разности потенциалов стационарного поля между теми двумя точками, к которым подключены проводники.
2. В отличие от электростатического поля, стационарное поле движущихся зарядов проникает внутрь проводника (дело в том, что свободные заряды, участвуя в направленном движении, не успевают должным образом перестраиваться и принимать «электростатические» конфигурации).
Линии напряжённости стационарного поля внутри проводника параллельны его поверхности, как бы ни изгибался проводник. Поэтому, как и в однородном электростатическом поле, справедлива формула , где — напряжение на концах проводника, — напряжённость стационарного поля в проводнике, — длина проводника.

Звоните нам: 8 (800) 775-06-82 (бесплатный звонок по России) +7 (495) 984-09-27 (бесплатный звонок по Москве)

Куда течёт ток? | ЗАО «МПО Электромонтаж»

В каком направлении течёт электрический ток в электрической цепи? Даже школьнику известно: во внешней цепи от плюса источника энергии к минусу, а внутри источника тока от минуса к плюсу.

Вспомним, однако: электрическим током в физике и электротехнике называется упорядоченное движение электрически заряженных частиц. Таковыми в металлических проводниках могут быть только отрицательно заряженные частицы — электроны, которые во внешней цепи движутся как раз наоборот: от минуса источника к плюсу. Получается, что за направление электрического тока в науке принимают направление противоположное существующему — движению электронов.

Такое парадоксальное положение электротехники как науки можно объяснить, обратившись к истории самой науки.

Среди множества концепций, которыми в старые времена пытались объяснить электрические явления, некоторые сегодня кажутся не вполне научными, но сыграли свою положительную роль.

Одну из них — унитарную теорию электричества — выдвинул американский ученый XVIII века Бенджамен Франклин. Он полагал, что электрическая материя представляет собой невесомую жидкость, которая содержится во всех телах и может вытекать из одних тел и накапливаться в других. Тела становятся наэлектризованными, и когда в них бывает её недостаток — это отрицательная электризация, а когда избыток — положительная. При соединении положительно заряженных тел с отрицательными электрическая жидкость переходит от тела с повышенным количеством жидкости к телам с пониженным количеством — как в сообщающихся сосудах.

Так Франклин ввёл понятия положительного и отрицательного зарядов и их движения, электрического тока, а англичанин Стефан Грэй обнаружил, что существуют такие вещества — металлы — которые проводят электричество от одного тела к другому.

Эти концепции предвосхитили электронную теорию проводимости.

Их современник, французский академик Шарль Франсуа Дюфе считал, что существует два вида электричества, они подчиняются каждое в отдельности теории Франклина, но при соприкосновении нейтрализуют друг друга.

Английский учёный Роберт Симмер, на основании опытов Дюфе и наблюдая за электризацией своих шелковых чулок, впервые в мире обнаружил, что заряжается не только натираемое, но и натирающее тело. То есть при трении тел друг о друга на каждом из них накапливаются заряды одного типа, причём заряды одного знака отталкиваются, а разного знака притягиваются друг к другу и компенсируются при соединении, делая тело нейтральным (незаряженным).

Дуалистическая теория стала основой для разработки ионной теории проводимости газов и растворов — после открытия явления электролиза, при котором были экспериментально установлены два противоположных направления движения зарядов — положительных — от плюса к минусу, и отрицательных — от минуса к плюсу.

В 1820 году датский учёный Ханс Христиан Эрстед открыл, что проводник с током влияет на показания магнитной стрелки, правда, сформулировал его несколько туманно: «полюс, который видит отрицательное электричество входящим над собой, отклоняется к востоку». В целях какой-то определённости в этих знаках и отклонениях член Парижской академии наук Андре-Мари Ампер предложил за основное условно принять направление одного из двух электричеств, а именно — положительное.

Почему он так решил? Возможно, потому, что упомянутый С. Грэй электропроводимость металлов уже установил, а вот обеспечивающий её отрицательно заряженный электрон английский физик Джон Джозеф Томсон открыл только в 1897 г.

Установивший существование электромагнитной индукции — наведение тока в проводнике в изменяющемся магнитном поле — Майкл Фарадей, между тем, писал: «Если я говорю, что ток идет от положительного места к отрицательному, то лишь в согласии с традиционным соглашением, заключённым между учеными — это обеспечивает постоянное ясное и определённое средство для указания направления сил этого тока».

Именно для ясного понимания и лёгкого запоминания физики электромагнитных явлений учёные — они же профессора — придумали мнемонические правила, известные нынешним школьникам и студентам как «правило левой руки» и «правило правой руки», которые, как бы для простоты, не стали отменять и после открытия реального носителя тока — электрона.

И всё бы ничего — но изобрели ещё и электронную лампу, в которой уж точно ток создаётся электронами, летящими из катода к положительно заряженному аноду. А для объяснения физических явлений в полупроводниковых приборах даже придумали виртуальный носитель положительного заряда — «дырку», то есть отсутствие электрона в молекуле, и предпочитают говорить не о направлениях тока, а о направлениях движения электронов и «дырок».

А в электротехнике всё ещё — вот уже полтораста лет — условные положительные заряды условно движутся от плюса к минусу. Можно бы, в интересах истины, поправить учебники, переписать монографии, переучить электриков. Это может вызвать путаницу и неудобства, во всяком случае, на первых порах. Но можно и не поправлять, потому что, как это обосновал американский физик и историк науки Томас Кун, всякое научное знание условно. В астрономии, например, Земля вращается вокруг солнца, а в метеорологии — Солнце вокруг земли. Физики считают законы Ньтона условными — частным случаем созданных ими двух теорий относительности — специальной и общей.

Может, и пусть остаётся как есть: от того, что мы изменим условное направление электрического тока, лампочки ярче не засветятся, мартены не погаснут, телевизоры задом-наперёд показывать не будут?

(Мы-то с вами знаем, куда течёт ток!).

Использована статья Б. Г.?Хасапова «История одного парадокса электротехники»

Что такое электрический ток

  

главная

основы

элементы

примеры расчетов

любительская технология

общая схемотехника

радиоприем

конструкции для дома и быта

связная аппаратура

телевидение

справочные данные

измерения

обзор радиолюбительских схем в журналах

обратная связь

       реклама 

 

С этой странички Вы можете по ссылкам попасть на страницы, описывающие основы электро-радиотехники.

что такое электрический ток   подготовка рабочего места   техника безопасности  немного о пайке

                    НЕМНОГО ОБ ЭЛЕКТРИЧЕСКОМ ТОКЕ

Представьте себе большой резервуар, в котором находится под давлением вода, готовая в любую минуту вырваться наружу. От резервуара отходит труба с краном. Открыли кран, и вода устремилась через трубу, например, в бассейн. Если диаметр трубы мал, скорость потока небольшая. Увеличили диаметр трубы — возросла и скорость потока. Происходит так потому, что с увеличением диаметра труба оказывает меньшее сопротивление напору воды, и вода вытекает с большей скоростью.
Допустим, что резервуар с водой — это источник электрической энергии, обладающей определенным напряжением (давлением воды), а труба — нагрузка, сопротивление (диаметр трубы) которой может изменяться. Тогда водный поток можно принять за электрический ток, протекающий через нагрузку.
Пока сопротивление нагрузки мало (диаметр трубы большой), через нее течет значительный ток (большая скорость потока). Когда же сопротивление возрастает (уменьшается диаметр трубы), электрический ток (скорость потока), наоборот, падает. С помощью этой аналогии вы, наверное, можете самостоятельно определить, как изменится ток при увеличении напряжения (повышении давления воды в резервуаре).
А теперь перейдем к единицам измерения напряжения, тока и сопротивления. Напряжение измеряют в вольтах, обозначая эту единицу буквой В. Если вы посмотрите на этикетку плоской батареи от карманного фонаря, то заметите на ней надпись «4,5 В». Это значит, что напряжение батареи 4,5 Вольта. На этикетке круглой батареи (правильнее ее называть элементом) уже другая цифра — 1,5 В, то есть напряжение ее 1,5 В.
И еще на этикетке есть знаки » + » и » - «. Это полярность выводов. Она указывает, в каком направлении будет течь ток, если к батарее подключить нагрузку, скажем лампочку от карманного фонаря. Вы все, конечно, видели такую лампочку и знаете, что внутри стеклянного баллона в ней подвешен тонкий металлический волосок. Один конец его припаян к резьбовой части лампочки, а другой — к контакту внизу. Резьбовая часть и контакт — это выводы лампочки. Как только они оказываются подключенными к выводам батареи, через нить лампочки начинает течь электрический ток. Направление его будет определенным — от плюсового вывода батареи к минусовому. Поскольку ток течет постоянно в одном направлении, его называют постоянным, напряжение тоже постоянным.
У гальванических и аккумуляторных батарей и элементов есть еще один параметр — это ЭЛЕКТРИЧЕСКАЯ ЕМКОСТЬ. Электрическая емкость показывает, в течении какого времени источник может давать определенный ток. Электрическую емкость измеряют в АМПЕР-ЧАСАХ. 

ДЛЯ ПРИМЕРА: солевой гальванический элемент типа 316, или «АА» имеет (в зависимости от производителя и технологии изготовления) электрическую емкость около 0,6 а\ч. Это означает, что данный элемент может отдавать ток в 60 миллиампер в течении 10 часов непрерывно (600 ма\ч делить на 60 ма). Если рабочий ток элемента уменьшить до 10 миллиампер, то время непрерывной работы элемента увеличится до 60 часов. Теперь, по истечении 60 часов напряжение на элементе под нагрузкой упадет до 0,8 вольта (при начальном напряжении в 1,5 в). У щелочных  (алкалиновых) гальванических элементов электрическая емкость больше, чем у солевых в несколько (до 5) раз. Соответственно у них и выше стоимость…  Ток 10-часового разряда считается оптимальным для большинства элементов и аккумуляторов.

Если от элемента брать ток небольшими «порциями» (например - включать фонарик каждый час на несколько минут),  общее время работы (а значит — и емкость!) элемента возрастут. Происходит это из-за частичного восстановления активных веществ внутри элемента (так называемый «эффект самовосстановления»). Этот эффект наблюдается только у свежеизготовленных элементов и батарей. У полностью разряженных элементов внутри еще остается большое количество неотработанных компонентов, поэтому (в крайнем экстренном случае!) работу такого элемента можно несколько восстановить. Простейшим из способов восстановления является нагрев элемента примерно до 50 градусов, другим способом является механическое воздействие на корпус такого элемента (можно корпус слегка «прокатать» между двумя дощечками). Второй способ менее пригоден, так как корпус элемента может разрушиться, что вызовет вытекание электролита в питаемую конструкцию (и тем самым — может вызвать порчу!), но его также можно использовать в особых экстренных случаях. Главное — не «перестараться»! Третий из способов заключается в зарядке элемента постоянным током (как аккумулятора). Некоторые из элементов допускают до 10-15 циклов «заряд — разряд», но такое восстановление также может вызвать разрушение (взрыв!) корпуса элемента, поэтому производители пишут на этикетке элемента фразу «не перезаряжать — опасно!». Используйте этот способ на свой страх и риск…
Во время хранения гальванических элементов происходит их саморазряд (обусловлен рядом причин), что понижает емкость элементов и батарей. Снижение емкости происходит, в основном, за счет необратимых химических процессов, происходящих во время хранения. Обычно на элементах и батареях указывают конечный срок хранения, но часто элемент продолжает работать и после окончания срока хранения (иногда - и наоборот!) — все зависит от условий хранения и добросовестности изготовителя. Если элемент хранить в сухом, прохладном месте (считается, что температура хранения около плюс 4 градусов Цельсия - самая оптимальная), срок его хранения может быть увеличен в несколько раз! По истечении какого-то времени, батарея может вообще потерять свою емкость. Для отбраковки таких элементов можно использовать измерительный прибор - ВОЛЬТМЕТР, но при этом необходимо обязательно подключать к элементу нагрузочный резистор! Дело в том, что вольтметр, обладая высоким внутренним сопротивлением, практически не нагружает элемент и напряжение на выводах элемента будет мало отличаться от указанного на этикетке. Сопротивление такого резистора должно быть таким, чтобы при измерении нагрузить элемент или батарею номинальным током. Так, для примера, элемент типоразмера «D» имеет емкость около 3,5 ампер/часов, следовательно его при измерении нужно нагрузить резистором сопротивлением около 5 ом (можно просто использовать электролампочку от карманного фонаря)…
«А почему не указывают полярность на гнездах сетевой розетки?» — спросите вы. Дело в том, что сетевое напряжение переменное. То в одном гнезде розетки плюс напряжения, в другом — минус, то наоборот. Такая смена полярности происходит 100 раз в секунду. При включении в розетку, например, настольной лампы, через ее нить потечет ток, направление которого будет меняться столько же раз в секунду, сколько и полярность напряжения.
Электрический ток измеряют в амперах, обозначая эту единицу буквой А. Но в практике радиолюбителя с такими токами встречаются редко, поэтому пользуются более мелкой единицей измерения - миллиампером — тысячной долей ампера, обозначаемой буквами мА, либо микроампером — одной тысячной долей миллиампера, обозначаемой буквами Мка.
Сопротивление измеряют в омах (условное обозначение Ом). Кроме этой единицы, используются более крупные: килоом (1 кОм = 1000 Ом) и мегаом (1 МОм=1000 кОм=1 000 000 Ом).
В практике конструирования электронных устройств вам неоднократно придется проводить различные расчеты электрических цепей. 
Для большинства расчетов пользуются законом Ома. Это основной  закон всей радиотехники : 

«СИЛА ТОКА В ЭЛЕКТРИЧЕСКОЙ ЦЕПИ ПРЯМО ПРОПОРЦИОНАЛЬНА ПРИЛОЖЕННОМУ НАПРЯЖЕНИЮ И ОБРАТНО ПРОПОРЦИОНАЛЬНА СОПРОТИВЛЕНИЮ ЭТОЙ ЦЕПИ«. 

Как это расшифровать? Ток в цепи тем больше, чем больше напряжение. При увеличении сопротивления в цепи (при неизменном напряжении) ток уменьшается. И наоборот.
Для практических целей достаточно запомнить вот этот треугольник:

  

Сила тока обозначается латинской буквой I, напряжение — U и сопротивление — R.

Второй закон электрической цепи называется Законом Кирхгофа, который гласит:

«В ЛЮБОМ УЗЛЕ ЭЛЕКТРИЧЕСКОЙ ЦЕПИ СУММА ПРИТЕКАЮЩИХ ТОКОВ РАВНА СУММЕ ВЫТЕКАЮЩИХ ТОКОВ«.

Например, если взять какой либо узел (соединение нескольких проводов или элементов в одной точке), то сила тока, входящего в этот узел будет равна силе тока, выходящей из этого узла. Для упрощения на рисунке показан простейший узел электрической цепи. Вытекающих токов может быть сколько угодно…

На рисунке I1 - притекающий, I2, I3 вытекающие токи.

Если вас интересует тема химических источников тока — вы можете более подробно  узнать об этом  по этой ссылке,  либо скачать книжку здесь.

Очень хорошую книжку В.В.Фролова «Язык радиосхем» (около 3.5 мегабайт) можно скачать по этой ссылке.

Для начинающего радиолюбителя будет полезна книжка «Первые шаги в радиоэлектронике» Болгарского автора Шишкова, скачать которую (около 3 мегабайт) можно здесь.

Отличную, на мой взгляд, книжку «Радиоэлектроника. Шаг за шагом» написал автор Рудольф Сворень. Скачать эту книжку (ВНИМАНИЕ!  Размер файла — около 20 мегабайт!) можно по ссылке.

                                                      вверх

Урок 3. магнитная индукция. действие магнитного поля на проводник с током и движущуюся заряженную частицу — Физика — 11 класс

Физика, 11 класс

Урок 3. Магнитная индукция. Действие магнитного поля на проводник и движущуюся заряжённую частицу

Перечень вопросов, рассматриваемых на уроке:

1) магнитное поле;

2) вектор магнитной индукции, линии магнитной индукции;

3) сила Ампера, сила Лоренца;

4) правило буравчика, правило левой руки.

Глоссарий по теме

Магнитная индукция – векторная величина, характеризующая величину и направление магнитного поля.

Сила Ампера – сила, действующая со стороны магнитного поля на проводник с током.

Сила Лоренца – сила, действующая со стороны магнитного поля на движущую частицу с зарядом.

Правило «буравчика» — правило для определения направления магнитного поля проводника с током.

Правило левой руки – правило для определения направления силы Ампера и силы Лоренца.

Соленоид – проволочная катушка.

Рамка с током – небольшой длины катушка с двумя выводами из скрученного гибкого проводника с током, способная поворачиваться вокруг оси, проходящей через диаметр катушки.

Основная и дополнительная литература по теме урока

Мякишев Г.Я., Буховцев Б.Б.,. Чаругин В.М. Физика.11 класс. Учебник для общеобразовательных организаций. М.: Просвещение, 2014. – С. 3 – 20

2. А.П. Рымкевич. Сборник задач по физике. 10-11 классы. — М: Дрофа, 2009. – С.109 — 112

Основное содержание урока

Магнитное поле – особый вид материи, которая создаётся электрическим током или постоянными магнитами. Для демонстрации действия и доказательства существования магнитного поля служат магнитная стрелка, способная вращаться на оси, или небольшая рамка (или катушка) с током, подвешенная на тонких скрученных гибких проводах.

Рамка с током и магнитная стрелка под действием магнитного поля поворачиваются так, что северный полюс (синяя часть) стрелки и положительная нормаль рамки указывают направление магнитного поля.

Магнитное поле, созданное постоянным магнитом или проводником с током, занимает всё пространство в окрестности этих тел. Магнитное поле принято (удобно) изображать в виде линий, которые называются линиями магнитного поля. Магнитные линии имеют вихревой характер, т.е. линии не имеют ни начала, ни конца, т.е. замкнуты. Направление касательной в каждой точке линии совпадает с направлением вектора магнитной индукции. Поля с замкнутыми линиями называются вихревыми.

Магнитное поле характеризуется векторной величиной, называемой магнитной индукцией. Магнитная индукция характеризует «силу» и направление магнитного поля – это количественная характеристика магнитного поля.

Она обозначается символом За направление вектора магнитной индукции принимают направление от южного полюса к северному магнитной стрелки, свободно установившейся в магнитном поле.

Направление магнитного поля устанавливают с помощью вектора магнитной индукции.

Направление вектора магнитной индукции прямого провода с током определяют по правилу буравчика (или правого винта).

Правило буравчика звучит следующим образом:

если направление поступательного движения буравчика совпадает с направлением тока в проводнике, то направление вращения ручки буравчика совпадает с направлением линий магнитного поля тока.

Направление магнитного поля внутри соленоида определяют по правилу правой руки.

Определим модуль вектора магнитной индукции.

Наблюдения показывают, что максимальное значение силы, действующей на проводник, прямо пропорционально силе тока, длине проводника, находящегося в магнитном поле.

F_max ~ I; F ~ Δl.

Тогда, зависимость силы от этих двух величин выглядит следующим образом

Отношение зависит только от магнитного поля и может быть принята за характеристику магнитного поля в данной точке.

Величина, численно равная отношению максимальной силы, действующей на проводник с током, на произведение силы тока и длины проводника, называется модулем вектора магнитной индукции:

Единицей измерения магнитной индукции является 1 тесла (Тл).

1Тл = 1Н/(1А∙1м).

Закон Ампера:

Сила, действующая на проводник с током в магнитном поле, равна произведению модуля магнитной индукции, силы тока, длины проводника и синуса угла между вектором магнитной индукции и направлением тока:

где α – угол между вектором B и направлением тока.

Направление силы Ампера определяется правилом левой руки:

Если ладонь левой руки развернуть так, чтобы линии магнитной индукции входили в ладонь, а четыре вытянутых пальца были направлены по направлению тока, то отогнутый на 900 большой палец покажет направление силы Ампера.

Сила Ампера — сила, действующая на проводник с током со стороны магнитного поля.

Сила Лоренца – сила, действующая на движущуюся заряженную частицу со стороны магнитного поля. Её численное значение равно произведению заряда частицы на модули скорости и магнитной индукции и синус угла меду векторами скорости и магнитной индукции:

– заряд частицы;

– скорость частицы;

B – модуль магнитной индукции;

– угол между векторами скорости частицы и магнитной индукции.

Направление силы Лоренца также определяют по правилу левой руки:

Если четыре вытянутых пальца левой руки направлены вдоль вектора скорости заряженной частицы, а вектор магнитной индукции направлен в ладонь, то отведённый на 900 большой палец покажет направление силы Лоренца. Если частица имеет заряд отрицательного знака, то направление силы Лоренца противоположно тому направлению, которое имела бы положительная частица.

Получим формулы для радиуса окружности и периода вращения частицы, которая влетает в однородное магнитное поле перпендикулярно линиям магнитной индукции, применяя формулы второго закона Ньютона и центростремительного ускорения.

Согласно 2-му закону Ньютона

Отсюда

Время, за которое частица делает полный оборот (период обращения), равно:

Многим юным бывает досадно, что они не родились в старые времена, когда делались открытия. Им кажется, что теперь всё известно и никаких открытий на их долю не осталось.

Одной из нераскрытых тайн является механизм земного магнитного поля. Как же и чем вызывается магнитное поле Земли? Подумайте и может быть…

Одна из возможных гипотез.

Как известно, ядро Земли имеет высокую температуру

и высокую плотность. Судя по исследованиям, в самом центре содержится твёрдое ядро. При вращении Земли вокруг своей оси центр тяжести не совпадает с геометрическим центром из-за притяжения Солнца. В результате сместившееся из центра ядро вращаясь относительно оболочки Земли вызывает такое же движение жидкой расплавленной массы мантии, как чайная ложка, перемешивающая воду в стакане. Получается не что иное, как направленное движение зарядов. Есть электрический ток, а он, в свою очередь, создаёт магнитное поле.

Разбор тренировочных заданий

1. На рисунке изображён проводник с током, помещённый в магнитное поле. Стрелка указывает направление тока в проводнике. Вектор магнитной индукции направлен перпендикулярно плоскости рисунка к нам. Как направлена сила, действующая на проводник с током?

Варианты ответов:

1. вправо →;

2. влево ←;

3. вниз ↓;

4. вверх ↑.

— точка означает, что магнитная индукция направлена на нас из глубины плоскости рисунка.

Используя правило левой руки, определяем направление силы Ампера:

Левую руку располагаем так, чтобы линии магнитной индукции входили в ладонь, 4 пальца направим вниз по направлению тока, тогда отогнутый на 900 большой палец покажет направление силы Ампера, т. е. она направлена влево.

Правильный вариант:

2. влево ←.

2. По проводнику длиной 40 см протекает ток силой 10 А. Чему равна индукция магнитного поля, в которое помещён проводник, если на проводник действует сила 8 мН?

(Ответ выразите в мТл).

3. Определите модуль силы, действующей на проводник длиной 50 см при силе тока 10 А в магнитном поле с индукцией 0,15 Тл. (Ответ выразите в мН).

4. Протон в магнитном поле с индукцией 0,01 Тл описал окружность радиусом 10 см. Найдите скорость протона. (Ответ выразите в км/с, округлив до десятков)

5. С какой скоростью влетает электрон в однородное магнитное поле (индукция 1,8 Тл) перпендикулярно к линиям индукции, если магнитное поле действует на него с силой 3,6∙10¹² Н? Ответ выразите в км/с.

6. Электрон движется в однородном магнитном поле с индукцией 3,14мТл. Чему равен период обращения электрона? (Ответ выразите в наносекундах, округлив до целых)

2. Дано:

l = 40cм = 0,4 м,

I = 10 A,

F =8 мН = 0,008 Н.

Найти: B

Решение:

Запишем формулу модуля магнитной индукции:

Делаем расчёт:

B = 0,008 Н / ( 0,4м·10 A) = 0,002 Tл = 2 мTл.

Ответ: 2 мTл.

3. Дано:

l = 50 cм = 0,5 м,

I = 10 A,

B = 0,l5 Tл.

Найти: F

Решение:

Запишем формулу силы Ампера:

Делаем расчёт:

F = 0,l5 Tл· 10 A· 0,5 м = 0,75 Н = 750 мН

Ответ: 750 мН.

4. Дано:

B = 0,0l Tл,

r = l0 cм = 0,l м.

Найти: v

Решение:

Заряд протона равен: q₀ = l,6·l0⁻ˡ⁹ Кл,

масса протона: m = l,67·l0⁻²⁷ кг.

Согласно 2-му закону Ньютона:

Отсюда следует:

Делаем расчёт:

v = ( l,6·l0⁻ˡ⁹ Кл·0,l м·0,0l Tл) / l,67·l0⁻²⁷ кг ≈ 0,00096·l0⁸ м/с ≈ l00 км/с.

Ответ: v ≈ l00 км/с.

5. Дано:

B = l,8 Tл,

F = 3,6·l0⁻¹² Н,

α = 90°.

Найти:

Решение:

Заряд электрона равен: q₀ = l,6·l0⁻ˡ⁹ Кл.

Используем формулу силы Лоренца:

.

Выразим из формулы силы скорость, учитывая, что sin90°=l,

Делаем расчёт:

v = 3,6·l0⁻¹² Н / (l,6·l0⁻ˡ⁹ Кл· l,8 Tл) = l,25·l0⁷м/с = l2500 км/с.

Ответ: v = l2500 км/с.

6. Дано:

B = 3,l4 мТл = 3,l4·l0⁻³ Tл,

q₀ = l,6·l0⁻ˡ⁹ Кл,

Найти: Т

Решение:

Масса электрона равна: m = 9,l·l0⁻³¹ кг.

Время, за которое частица делает полный оборот (период обращения), равно:

Делаем расчёт:

T = 2·3,l4·9,l·l0⁻³¹ кг/( l,6·l0⁻ˡ⁹ Кл·3,l4·l0⁻³ Tл) = ll,375·l0⁻⁹ с ≈ ll нс.

Ответ: T ≈ ll нс.

Учебное пособие по физике: электрический ток

Если два требования электрической цепи выполнены, заряд будет проходить через внешнюю цепь. Говорят, что есть ток — поток заряда. Использование слова current в этом контексте означает просто использовать его, чтобы сказать, что что-то происходит в проводах — заряд движется. Однако ток — это физическая величина, которую можно измерить и выразить численно. Как физическая величина, , ток — это скорость, с которой заряд проходит через точку в цепи.Как показано на диаграмме ниже, ток в цепи можно определить, если можно измерить количество заряда Q , проходящего через поперечное сечение провода за время t . Ток — это просто соотношение количества заряда и времени.

Текущее — это величина ставки. В физике есть несколько скоростных величин. Например, скорость — это величина скорости — скорость, с которой объект меняет свое положение. Математически скорость — это отношение изменения положения к времени.Ускорение — это величина скорости — скорость, с которой объект меняет свою скорость. Математически ускорение — это отношение изменения скорости к времени. А мощность — это величина скорости — скорость, с которой работа выполняется на объекте. Математически мощность — это отношение работы к времени. В каждом случае величины скорости математическое уравнение включает некоторую величину во времени. Таким образом, ток как величина скорости будет математически выражен как

Обратите внимание, что в приведенном выше уравнении для обозначения величины тока используется символ I .

Как обычно, когда количество вводится в Физическом классе, также вводится стандартная метрическая единица, используемая для выражения этой величины. Стандартная метрическая единица измерения тока — ампер . Ампер часто сокращается до Ампер и обозначается символом А . Ток в 1 ампер означает, что 1 кулон заряда проходит через поперечное сечение провода каждую 1 секунду.

1 ампер = 1 кулон / 1 секунда

Чтобы проверить свое понимание, определите ток для следующих двух ситуаций.Обратите внимание, что в каждой ситуации дается некоторая посторонняя информация. Нажмите кнопку Проверить ответ , чтобы убедиться, что вы правы.

Провод изолируют поперечным сечением 2 мм и определяют, что заряд 20 C может пройти через него за 40 с.

Сечение провода длиной 1 мм изолируется, и определяется, что заряд 2 Кл проходит через него за 0,5 с.

I = _____ Ампер

I = _____ Ампер

Обычное направление тока

Частицы, которые переносят заряд по проводам в цепи, являются подвижными электронами.Направление электрического поля в цепи по определению является направлением, в котором проталкиваются положительные испытательные заряды. Таким образом, эти отрицательно заряженные электроны движутся в направлении, противоположном электрическому полю. Но в то время как электроны являются носителями заряда в металлических проводах, носителями заряда в других цепях могут быть положительные заряды, отрицательные заряды или и то, и другое. Фактически, носители заряда в полупроводниках, уличных фонарях и люминесцентных лампах одновременно являются как положительными, так и отрицательными зарядами, движущимися в противоположных направлениях.

Бен Франклин, проводивший обширные научные исследования статического и токового электричества, считал положительные заряды носителями заряда. Таким образом, раннее соглашение о направлении электрического тока было установлено в том направлении, в котором будут двигаться положительные заряды. Это соглашение прижилось и используется до сих пор. Направление электрического тока условно является направлением движения положительного заряда. Таким образом, ток во внешней цепи направлен от положительной клеммы к отрицательной клемме батареи.Электроны действительно будут двигаться по проводам в противоположном направлении. Зная, что настоящими носителями заряда в проводах являются отрицательно заряженные электроны, это соглашение может показаться немного странным и устаревшим. Тем не менее, это соглашение, которое используется во всем мире, и к которому студент-физик может легко привыкнуть.

Зависимость тока от скорости дрейфа

Ток связан с количеством кулонов заряда, которые проходят точку в цепи за единицу времени.Из-за своего определения его часто путают со скоростью дрейфа количества. Скорость дрейфа означает среднее расстояние, пройденное носителем заряда за единицу времени. Как и скорость любого объекта, скорость дрейфа электрона, движущегося по проводу, — это отношение расстояния ко времени. Путь типичного электрона через проволоку можно описать как довольно хаотический зигзагообразный путь, характеризующийся столкновениями с неподвижными атомами. Каждое столкновение приводит к изменению направления электрона.Однако из-за столкновений с атомами в твердой сети металлического проводника на каждые три шага вперед приходится два шага назад. С электрическим потенциалом, установленным на двух концах цепи, электрон продолжает движение до , перемещаясь вперед на . Прогресс всегда идет к положительному полюсу. Однако общий эффект бесчисленных столкновений и высоких скоростей между столкновениями состоит в том, что общая скорость дрейфа электрона в цепи ненормально мала. Типичная скорость дрейфа может составлять 1 метр в час.Это медленно!

Тогда можно спросить: как может быть ток порядка 1 или 2 ампер в цепи, если скорость дрейфа составляет всего около 1 метра в час? Ответ таков: существует много-много носителей заряда, движущихся одновременно по всей длине цепи. Ток — это скорость, с которой заряд пересекает точку в цепи. Сильный ток является результатом нескольких кулонов заряда, пересекающих поперечное сечение провода в цепи. Если носители заряда плотно упакованы в провод, тогда не обязательно должна быть высокая скорость, чтобы иметь большой ток.То есть носители заряда не должны преодолевать большое расстояние за секунду, их просто должно быть много, проходящих через поперечное сечение. Ток не имеет отношения к тому, насколько далеко заряды перемещаются за секунду, а скорее к тому, сколько зарядов проходит через поперечное сечение провода в цепи.

Чтобы проиллюстрировать, насколько плотно упакованы носители заряда, мы рассмотрим типичный провод, который используется в цепях домашнего освещения — медный провод 14-го калибра. В срезе этой проволоки длиной 0,01 см (очень тонком) их будет целых 3.51 x 10 20 атом меди. Каждый атом меди имеет 29 электронов; маловероятно, что даже 11 валентных электронов одновременно будут двигаться как носители заряда. Если мы предположим, что каждый атом меди вносит только один электрон, то на тонком 0,01-сантиметровом проводе будет целых 56 кулонов заряда. При таком большом количестве подвижного заряда в таком маленьком пространстве малая скорость дрейфа может привести к очень большому току.

Чтобы проиллюстрировать это различие между скоростью заноса и течением, рассмотрим аналогию с гонками.Предположим, что была очень большая гонка черепах с миллионами и миллионами черепах на очень широкой гоночной трассе. Черепахи не очень быстро двигаются — у них очень низкая скорость дрейф . Предположим, что гонка была довольно короткой — скажем, длиной 1 метр — и что значительный процент черепах достиг финишной черты в одно и то же время — через 30 минут после начала гонки. В таком случае течение будет очень большим — миллионы черепах пересекают точку за короткий промежуток времени.В этой аналогии скорость связана с тем, насколько далеко черепахи перемещаются за определенный промежуток времени; а ток зависит от того, сколько черепах пересекли финишную черту за определенное время.

Природа потока заряда

Как только было установлено, что средняя скорость дрейфа электрона очень и очень мала, вскоре возникает вопрос: почему свет в комнате или в фонарике загорается сразу после включения переключателя? Разве не будет заметной задержки перед тем, как носитель заряда перейдет от переключателя к нити накала лампочки? Ответ — нет! и объяснение того, почему раскрывает значительную информацию о природе потока заряда в цепи.

Как было сказано выше, носителями заряда в проводах электрических цепей являются электроны. Эти электроны просто поставляются атомами меди (или любого другого материала, из которого сделана проволока) внутри металлической проволоки. Как только переключатель переводится в положение на , цепь замыкается, и на двух концах внешней цепи устанавливается разность электрических потенциалов. Сигнал электрического поля распространяется почти со скоростью света ко всем мобильным электронам в цепи, приказывая им начать марш с маршем .По получении сигнала электроны начинают двигаться по зигзагообразной траектории в обычном направлении. Таким образом, щелчок переключателя вызывает немедленную реакцию во всех частях схемы, заставляя носители заряда повсюду двигаться в одном и том же направлении. В то время как фактическое движение носителей заряда происходит с низкой скоростью, сигнал, который информирует о начале движения, движется со скоростью, составляющей долю от скорости света.

Электроны, которые зажигают лампочку в фонарике, не должны сначала пройти от переключателя через 10 см провода к нити накала.Скорее, электроны, которые зажигают лампочку сразу после того, как переключатель переведен в положение на , являются электронами, которые присутствуют в самой нити накала. Когда переключатель повернут, все подвижные электроны повсюду начинают движение; и именно подвижные электроны, присутствующие в нити накала, непосредственно ответственны за зажигание ее колбы. Когда эти электроны покидают нить накала, в нее входят новые электроны, которые ответственны за зажигание лампы. Электроны движутся вместе, как вода в трубах дома.Когда кран поворачивается с на , вода в кране выходит из крана. Не нужно долго ждать, пока вода из точки входа в ваш дом пройдет по трубам к крану. Трубы уже заполнены водой, и вода во всем водном контуре одновременно приводится в движение.

Развиваемая здесь картина потока заряда представляет собой картину, в которой носители заряда подобны солдатам, идущим вместе, повсюду с одинаковой скоростью.Их движение начинается немедленно в ответ на установление электрического потенциала на двух концах цепи. В электрической цепи нет места, где носители заряда расходуются или расходуются. Хотя энергия, которой обладает заряд, может быть израсходована (или лучше сказать, что электрическая энергия преобразуется в другие формы энергии), сами носители заряда не распадаются, не исчезают или иным образом не удаляются из схема. И нет места в цепи, где бы носители заряда начали скапливаться или накапливаться.Скорость, с которой заряд входит во внешнюю цепь на одном конце, такая же, как скорость, с которой заряд выходит из внешней цепи на другом конце. Ток — скорость потока заряда — везде одинакова. Поток заряда подобен движению солдат, идущих вместе, повсюду с одинаковой скоростью.

Проверьте свое понимание

1.Говорят, что ток существует всякий раз, когда _____.

а. провод заряжен

г. аккумулятор присутствует

г. электрические заряды несбалансированные

г. электрические заряды движутся по петле

2. У тока есть направление. По соглашению ток идет в направлении ___.

а. + заряды перемещаются

г.- электроны движутся

г. + движение электронов

3. Скорость дрейфа подвижных носителей заряда в электрических цепях ____.

а. очень быстро; меньше, но очень близко к скорости света

г. быстрый; быстрее, чем самая быстрая машина, но далеко не скорость света

г. медленный; медленнее Майкла Джексона пробегает 220-метровую

г.очень медленно; медленнее улитки

4. Если бы электрическую цепь можно было сравнить с водяной цепью в аквапарке, то ток был бы аналогичен ____.

Выбор:

A. давление воды

Б. галлонов воды, стекающей с горки в минуту

С.вода

D. нижняя часть ползуна

E. водяной насос

F. верх горки

5. На схеме справа изображен токопроводящий провод. Две площади поперечного сечения расположены на расстоянии 50 см друг от друга. Каждые 2,0 секунды через каждую из этих областей проходит заряд 10 ° C.Сила тока в этом проводе ____ А.

а. 0,10

г. 0,25

г. 0,50

г. 1.0

e. 5,0

ф. 20

г. 10

ч.40

и. ни один из этих

6. Используйте диаграмму справа, чтобы заполнить следующие утверждения:

а. Ток в один ампер — это поток заряда со скоростью _______ кулонов в секунду.

г. Когда заряд 8 Кл проходит через любую точку цепи за 2 секунды, ток составляет ________ А.

г. Если за 10 секунд поток заряда через точку A (диаграмма справа) проходит 5 ° C, то ток равен _________ A.

г. Если ток в точке D равен 2,0 А, то _______ C заряда проходит через точку D за 10 секунд.

e. Если 12 ° C заряда пройдет мимо точки A за 3 секунды, то 8 C заряда пройдут мимо точки E за ________ секунд.

ф. Верно или неверно:

Ток в точке E значительно меньше тока в точке A, поскольку в лампочках расходуется заряд.

Направление электрического тока

Направление электрического тока может немного сбивать с толку. Здесь мы ответили на ваш запрос. Надеюсь, эта статья поможет вам понять направление течения тока.

Электрический ток

Каждая частица в природе, кроме изоляторов *, имеет в себе большое количество свободных электронов.Эти электроны беспорядочно перемещаются во всех направлениях внутри материала при нормальных условиях. Если к этим материалам приложить определенное напряжение, все эти электроны начнут перемещаться из области с более высоким потенциалом в область с более низким потенциалом. Это движение электронов из области с более высоким потенциалом в область с более низким потенциалом под действием электрического поля составляет электрический ток.

* в изоляторе не будет свободных электронов при условии, что он поддерживается при нормальной или комнатной температуре.

Определение тока

Электрический ток обычно называют потоком зарядов по проводнику. Его можно определить как — количество заряда , которое проходит через площадь поперечного сечения в проводнике. Другими словами, термин «ток» можно определить как скорость прохождения зарядов через проводник. Узнать больше о Electic Current

Математическое выражение электрического тока

Электрический ток измеряется количеством электронов, проходящих через определенную точку в проводнике или цепи за единицу времени.

I = Q / т

Где Q — заряд электронов, протекающих по проводнику. t — время истечения в секундах.

В каком направлении течет электрический ток?

Направление электрического тока немного сложно понять тем, кого учили, что ток течет от положительного к отрицательному. За этим явлением стоят две теории. Одна из них — это теория обычного тока, а другая — теория фактического протекания тока.Когда Бенджамин Франклин изучал заряды, структура атома и атомных частиц была неизвестна. Следовательно, он принял точку накопления заряда как положительную, а точку, в которой отсутствуют заряды, как отрицательную. Следовательно, говорят, что заряд перетекает с положительного на отрицательный. Это называется обычным током.

Но на самом деле электрический ток — это не что иное, как поток электронов. Электроны — это отрицательно заряженные частицы, которые притягиваются к положительному заряду.Кроме того, многие эксперименты показали, что в проводнике текут свободные электроны. Отрицательно заряженные электроны перемещаются от отрицательной клеммы к положительной. Это направление фактического тока.

Направление тока в анализе цепей

С точки зрения схемотехнического анализа, мы обычно рассматриваем направление электрического тока от положительного к отрицательному. Математически отрицательный заряд, текущий в одном направлении, эквивалентен положительному заряду, текущему в противоположном направлении.Следовательно, это не имеет значения. Во время анализа цепи можно рассматривать движение тока от положительного к отрицательному или наоборот. Фактически, положительно заряженные ионы могут притягиваться отрицательно заряженными электронами.

Единица тока

Единица измерения тока — ампер или А. Один ампер равен одному кулону в секунду, тогда как один кулон равен 6,25 x 10 18 электронов. Говоря, что через цепь проходит один ампер тока, это означает, что 6.25 x 10 18 электронов пересекают точку в цепи за секунду.

Правило правой руки для токоведущего провода

Если вы считаете, что контент, доступный через Веб-сайт (как определено в наших Условиях обслуживания), нарушает или несколько ваших авторских прав, сообщите нам, отправив письменное уведомление («Уведомление о нарушении»), содержащее в информацию, описанную ниже, назначенному ниже агенту.Если репетиторы университета предпримут действия в ответ на ан Уведомление о нарушении, оно предпримет добросовестную попытку связаться со стороной, которая предоставила такой контент средствами самого последнего адреса электронной почты, если таковой имеется, предоставленного такой стороной Varsity Tutors.

Ваше Уведомление о нарушении прав может быть отправлено стороне, предоставившей доступ к контенту, или третьим лицам, таким как в виде ChillingEffects.org.

Обратите внимание, что вы будете нести ответственность за ущерб (включая расходы и гонорары адвокатам), если вы существенно искажать информацию о том, что продукт или действие нарушает ваши авторские права.Таким образом, если вы не уверены, что контент находится на Веб-сайте или по ссылке с него нарушает ваши авторские права, вам следует сначала обратиться к юристу.

Чтобы отправить уведомление, выполните следующие действия:

Вы должны включить следующее:

Физическая или электронная подпись правообладателя или лица, уполномоченного действовать от их имени; Идентификация авторских прав, которые, как утверждается, были нарушены; Описание характера и точного местонахождения контента, который, по вашему мнению, нарушает ваши авторские права, в \ достаточно подробностей, чтобы позволить репетиторам университетских школ найти и точно идентифицировать этот контент; например нам требуется а ссылка на конкретный вопрос (а не только на название вопроса), который содержит содержание и описание к какой конкретной части вопроса — изображению, ссылке, тексту и т. д. — относится ваша жалоба; Ваше имя, адрес, номер телефона и адрес электронной почты; а также Ваше заявление: (а) вы добросовестно считаете, что использование контента, который, по вашему мнению, нарушает ваши авторские права не разрешены законом, владельцем авторских прав или его агентом; (б) что все информация, содержащаяся в вашем Уведомлении о нарушении, является точной, и (c) под страхом наказания за лжесвидетельство, что вы либо владелец авторских прав, либо лицо, уполномоченное действовать от их имени.

Отправьте жалобу нашему уполномоченному агенту по адресу:

Чарльз Кон Varsity Tutors LLC
101 S. Hanley Rd, Suite 300
St. Louis, MO 63105

Или заполните форму ниже:

Условный ток относительно потока электронов

Условный ток относительно потока электронов © 2015 Chris E. ChaulkVTR1R2R3ITIT

Нажмите кнопку вверху.

Обычный ток предполагает, что ток течет от положительной клеммы, через цепь к отрицательной клемме источника.Это было условием, выбранным при открытии электричества. Они были не правы!

Электронный поток — это то, что происходит на самом деле, и электроны текут из отрицательной клеммы через цепь в положительную клемму источника.

Используются как обычный ток, так и поток электронов. Многие учебники доступны в обоих форматах.


Флойд, 1989, Принципы электрических цепей , 5-е издание, Версия для обычных токов.


Флойд, 1990, Принципы электрических цепей , 4-е издание, версия для электронного потока.

На самом деле, не имеет значения, в каком направлении протекает ток, пока он используется последовательно . Направление тока не влияет на его действия.

Как правило, программа Electron Flow используется в программах по физике средней школы и двухгодичных программах для технических специалистов.

Но трехлетние технологические и университетские инженерные программы используют обычный ток.Определенные символы (например, диоды и транзисторы) и правила (например, правила правой руки) были созданы с использованием обычного тока. Переход от обычного тока к электронному потоку вызовет некоторую путаницу для старых и новых студентов, и возникнут ошибки, поэтому традиционный ток был сохранен, чтобы не было путаницы с теми, кто уже обучался с обычным током. Две системы могут показаться сбивающими с толку, но пока их использование единообразно, на самом деле это не так!

Вы должны понимать, какое соглашение используется, потому что правила меняются.Бывший. Правила правой руки в обычном токе становятся правилами левой руки в электронном потоке. Пример

На протяжении всего курса используется обычный ток. Поэтому всегда предполагайте, что ток течет через положительный вывод источника.

ELTK1100

Собственный полупроводник — условный ток

Обычный текущий

Электрический ток, протекающий с положительной клеммы аккумулятора. к отрицательной клемме аккумулятора называется обычным Текущий.Обычное направление тока такое же направление потока отверстий, но против направления потока свободного электроны.

Когда Бен Франклин начал экспериментировать с электричество, он предположил, что электрический ток (положительный носители заряда) текут с положительного на отрицательный. Но позже мир понял, что это было неправильно. Фактически электрический ток течет от отрицательной клеммы к положительный полюс аккумуляторной батареи.

поток носителей заряда называется током. Здесь носители заряда протоны или электроны. Но именно ток переносится электронов, а не протонов, потому что протоны сильно связаны с ядро атома из-за сильного ядерная сила. Итак, электроны, которые слабо связаны с ядро атома разрывает связь с родительским атомом и стать свободным.Электроны, не связанные с атомами, текут свободно и представляют собой текущие. Электроны текут из отрицательного клемма к плюсовой клемме аккумуляторной батареи.

Большинство людей в мире подписаны на условное направление тока. В некоторых учебниках актуальны направление написано противоположно обычному току направление, т.е. рассматривая поток электронов как направление Текущий.Но в большинстве учебников используется обычный ток. направление. Большинство электронные устройства анализируются с использованием этого обычного текущее направление.

Это важно понимать разницу между обычными направление тока и направление тока электронного потока. Мы можем следовать любому. Неважно, используем ли мы обычное направление тока или направление потока электронов.Оба из них дадут тот же результат. Мы можем спроектировать схему используя любой из них.



Общие сведения об электричестве — Узнайте об электричестве, токе, напряжении и сопротивлении


Дом > Поддержка> Общие сведения об электричестве
Общие сведения об электричестве

Что такое электричество?

Любая бытовая техника, которую мы используем в нашей повседневной жизни, например, бытовая техника, оргтехника и промышленное оборудование, почти все это требует электричества.Следовательно, мы должны понимать электричество.

Первый вопрос, который мы узнаем ответ « где электричество родом из? «

Все дела состоят из атомы. Затем задайте следующий вопрос: « Что такое атомы? ».
Атомы — это самая маленькая часть элемента.Они состоят ядра и электронов, электроны окружают ядро. Элементы идентифицируются по количеству электронов на орбите вокруг ядра атомов и числом протонов в ядре.


Ядро состоит из протонов и нейтронов, а количество протоны и нейтроны уравновешены. У нейтронов нет электрического заряда, протоны имеют положительный заряд (+), а электроны — отрицательный заряды (-).Положительный заряд протона равен отрицательному заряду электрона.

Электроны связаны по своей орбите за счет притяжения протонов, но электроны во внешней зоне могут покинуть свою орбиту за счет некоторые внешние силы. Их называют свободными электронами, которые движутся от одного атома к другому, образуются потоки электронов. Это основа электричества. Материалы, позволяющие свободно перемещающиеся электроны называются проводниками а материалы, которые позволяют перемещаться небольшому количеству свободных электронов, называются изоляторы .

Все вещества состоят из атомов, имеющих электрические заряды. Следовательно, у них есть электрические заряды. Что касается сбалансированного количество протонов и электронов, сила положительного заряда и сила отрицательного заряда уравновешена. Это называется нейтральным состоянием. атома. (Число протонов и электронов остается равным.)

« Статическое электричество » представляет собой ситуацию, когда все вещи состоят из электрических обвинения.Например, трение материала о другой может вызвать статическое электричество. Свободные электроны одного материала двигаться с силой, пока они не освободятся от своих орбит вокруг ядра и перейти к другому. Электроны одного материала уменьшаются, он представляет собой положительный заряд. В то же время электроны другого увеличение, он имеет отрицательные заряды.

В общем, заряд производство материи означает, что материя имеет электрические заряды.Он имеет положительный и отрицательный заряды, что выражается в кулон.


Ток, Напряжение и сопротивление


Что сейчас?

Электрическое явление вызвано потоком свободные электроны от одного атома к другому.Характеристики из текущего электричества противоположны тем статического электричества.

Провода состоят из проводников, например медных. или алюминий. Атомы металла состоят из свободных электронов, которые свободно переходить от одного атома к другому. Если добавлен электрон в проводе свободный электрон притягивается к протону, чтобы оставаться нейтральным. Вытеснение электронов с их орбит может вызвать недостаток электронов.Электроны, которые непрерывно движутся по проволоке, называются Electric. Текущий .



Для одножильных проводов
электрический ток относится к направленным отрицательно-положительные электроны от одного атома к другому. Жидкость проводники и газопроводы, электрический ток относится к электронам а протоны текут в обратном направлении.

Ток — это поток электронов, но ток и электроны текут в противоположное направление. Ток течет от положительного к отрицательному и электроны перетекают с отрицательного на положительный.


Ток определяется количеством электронов, проходящих через сечение проводника за одну секунду. Ток измеряется в ампер , что сокращенно « ампер ».Обозначение усилителя — это буква « A ».

А ток в один ампер означает что ток проходит через поперечное сечение двух проводников, которые расположены параллельно на расстоянии 1 метра друг от друга с 2х10 -7 Ньютон сила на метр возникает в каждом проводнике. Это также может означать сборы одного кулона (или 6,24х10 18 электронов), проходящего через поперечное сечение проводника за одну секунду.


Что такое напряжение?

Электрический ток — это поток электронов в проводнике. Сила необходим для протекания тока через проводник, называется напряжение и потенциал другой срок напряжения. Например, у первого элемента больше положительные заряды, поэтому он имеет более высокий потенциал. С другой стороны, второй элемент имеет более отрицательные заряды, поэтому он имеет более низкий потенциал.Разница между двумя точками называется разность потенциалов .

Электродвижущая сила означает силу, которая заставляет ток непрерывно течь через дирижер. Эта сила может создаваться генератором энергии, аккумулятор, аккумулятор фонарика и топливный элемент и т. д.

Вольт, сокращенно « В », это единица измерения измерения, используемые взаимозаменяемо для напряжения, потенциала, и электродвижущая сила.Один вольт означает силу, которая заставляет ток в один ампер проходит через сопротивление в один Ом.

Что такое сопротивление?
Электроны движутся через проводник при протекании электрического тока. Все материалы мешают протекание электрического тока до некоторой степени. Эта характеристика называется сопротивлением .Сопротивление увеличивается с увеличением длины или уменьшением поперечного сечения материал.

Единица измерения сопротивления Ом и его символ — греческая буква омега ( Ω ). Сопротивление в один Ом означает, что проводник пропускает ток. одного ампера на поток с напряжением один вольт.

Все материалы различаются по пропусканию электронов.Материалы которые позволяют свободно перемещаться большому количеству электронов, называются проводниками такие как медь, серебро, алюминий, раствор хлористоводородной, серной кислота и соленая вода. Напротив, материалы, пропускающие мало электронов для протекания называются изоляторы типа пластмассовые, резина, стекло и сухая бумага. Другой тип материалов, полупроводники имеют характеристики как проводников, так и изоляторов.Они позволяют электронам двигаться, в то же время имея возможность контролировать поток электронами и примерами являются углерод, кремний, германий и т. д.

Сопротивление проводника зависит от следующих двух основных факторов:

1. Виды материала
2. Температура материала

Как измерить ток

Прибор для измерения силы тока называется амперметр или амперметр .
Шаги для измерения тока Подключите небольшую лампочку к сухой батарее.Измерьте ток который проходит через лампочку при подключении положительной клеммы (+) амперметра к отрицательной клемме (-) сухого элемента (см. рисунок)
Указания по технике безопасности при измерении силы тока;
1. Оценить ток, требующий измерения затем выберите подходящий амперметр, так как каждый амперметр имеет разные предел измерения тока.
2. Убедитесь, что соединение с плюсовой клеммой (+) и отрицательная клемма (-) амперметра правильные.
3. Не подключайте напрямую клеммы амперметра сушить клеммы ячеек. Так как это может повредить счетчик.

Как измерить напряжение
Прибор для измерения напряжения, разницы Потенциальная или электродвижущая сила называется вольтметром .

Шаги для измерения напряжения
Подключите небольшую лампочку к сухому элементу. Вольтметр есть подключен параллельно лампочке для измерения напряжения поперек лампочки. Подключите положительный вывод (+) вольтметр к плюсовой клемме (+) сухого элемента и подключите отрицательная клемма (-) вольтметра к отрицательной клемме (-) сухой ячейки (см. рисунок).
Указания по технике безопасности при измерении Напряжение;
1. Оценить напряжение, необходимое для измерения затем выберите подходящий вольтметр
, поскольку каждый вольтметр рассчитан на
предел измерения напряжения.
2. Убедитесь, что подключение положительной клеммы (+) и отрицательная клемма (-) вольтметра правильные.

Как измерить сопротивление
Инструмент, используемый для измерения Сопротивление называется тестером или мультиметром .Мультиметр или тестовый метр используется для изготовления различных электрических измерения, такие как ток, напряжение и сопротивление. Он сочетает в себе функции амперметра, вольтметра и омметра.

Шаги для измерения сопротивления
Поверните лицевую шкалу в положение для требуемого измерения, сопротивления, затем коснитесь обоих выводов мультиметра (см. рисунок 1) и отрегулируйте диапазон измерителя на 0 Ом.Трогать оба вывода измерителя к сопротивлению и возьмите чтение (см. рисунок 2).


Как работает электричество?

Электрический ток — это способность делать работу.Электрический ток можно преобразовать в тепло, мощность и магнетизм, чтобы назвать несколько.

Электрический ток классифицирован по своим функциям и трем основным типам:

1.

Теплоэнергетика

2.

Электрохимия

3.

Магнетизм


1. Тепло и энергия используется для производства тепла и электроэнергии.
Например, нихромовая токоведущая проволока. проволока имеет высокое сопротивление и выделяет тепло.Это применяется быть составной частью электрических духовок, тостеров, электрических утюгов и лампочки и др.

Эксперимент проводится путем измерения нагреть количество воды калориметром. Увеличьте напряжение на провод вариаком и подключите амперметр и вольтметр для измерения ток и напряжение.
Установите шкалу переменного тока, чтобы отрегулировать значение напряжения и тока нихромовая проволока и ток периодически пропускается и измерить количество тепла от нихромовой проволоки.Есть какие-то указания напряжения и тока. Если напряжение, ток и время увеличиваются, количество тепла также увеличится. Они выражаются отношение, как показано ниже.

Это называется Джоуля. закон . Количество тепла зависит от напряжения время тока и интервал времени.По закону Ома V (напряжение) = I (ток) x R (сопротивление), следовательно,

Количество тепла зависит от текущий квадрат, умноженный на сопротивление и интервал времени.

При пропускании тока через нихромовую проволоку в воде ток превращается в тепло, и температура повышается. Работу выполняет тепло, выделяемое в электрической цепи, которая называется Electric. мощность .

Измеряется электрическая мощность в ватт-часах (Втч), а количество тепла измеряется в калориях. (Cal).

Работа выполняется за счет выделяемого тепла в электрической цепи написано мощность, что означает что номинальная работа выполняется в цепи, когда ток 1 А с Применяется 1 Вольт, а его единица измерения — ватт.

2. Электрохимия

Например, когда ток проходит через хлорид натрия (NaCl), химическая реакция, называемая электролизом. имеет место. Применяется для производства электролиза, цинкования. и аккумулятор и т. д.


Эксперимент проводится путем пропитывания двух платиновых (Pt) пластин. в расплаве соли. Подключите батареи к двум платиновым пластинам, ток проходит через расплав соли и производит хлор пузыри вокруг положительной пластины (+) и пузырьки водорода вокруг отрицательной пластины (-), поскольку хлорид натрия составляет натрия (Na) и хлорида (Cl).Когда хлорид натрия тает в воде, элементы разделяются. Натрий имеет положительные заряды (+), а у хлора отрицательные заряды (-) и эти заряды называются ионами . Расплав соли имеет оба положительных заряда, называемые анодами , а отрицательные заряды называются катодами . Состояние разделенных элементов называется ионизация .Если соль растапливается водой, в растворе имеются ионы, называется раствор электролита . И если текущий проходит через раствор электролита, химическая реакция происходит электролиз.

3. Магнетизм

Пример данной электромонтажной работы — токоведущий проволока, возникают магнитные линии потока.Это применяется для производства электродвигатели, электрические трансформаторы и магнитофоны, пр.

Понимание смысла магнетизма:
Что такое магнетизм?

Составная формула магнита: Fe 3 O 4 . Все магниты обладают двумя характеристиками. Во-первых, они привлекают и держи железо.Вторичный, если свободно двигаться, как компас игла, они займут положение север-юг. Любые материалы Имеют такие характеристики, они называются магнит .

Характеристики магнита
Каждый магнит имеет два полюса, один северный полюс и один южный полюс.
Противоположные полюса притягиваются друг к другу, в то время как полюса отталкивают друг друга.

Электричество и магнитное поле

Когда магнитная стрелка находится рядом с электрическим проводом, который ток пропускается, магнитная стрелка включает направление тока (см. рисунок 1 и 2).Следовательно, электрический ток также создает связанный магнитный силу или говорят, что электричество способно производить магнитное поле.

Когда магнитная игла помещена в катушку проволоки с одной петлей (см. рисунок), и ток проходит через катушку с проволокой, магнитный игла поворачивается в направлении, показанном на рисунке выше.А направления магнитных линий потока показаны стрелки.

Когда магнитная игла помещена в проволочную катушку с множеством петель как показано на правом рисунке, ток проходит через катушка. Направление магнитных линий магнитных параллелей катушка проволоки. Характеристики магнитных линий потока как характеристики магнита, но без магнитного полюса.

Когда катушка с токоведущим проводом помещается рядом с железным стержнем, железный стержень немного сдвинется (см. рисунок 1). Если сердечник размещен в катушке из проволоки железный стержень сильно притягивается (см. фигура 2). Поскольку сердечник — это мягкое железо, которое проводит магнитные силовые линии, когда ток проходит через проволочную катушку вокруг сердечника сердечник намагничивается с высокой мощностью что называется электромагнитов .Эта функция широко применяется в промышленности.

Магнитное поле, связанное с током | Электромагнетизм

10.2 Магнитное поле, связанное с током (ESBPS)

Если вы поднесете компас к проводу, через который проходит ток. течет, стрелка компаса отклоняется.

Поскольку компасы работают, указывая вдоль силовых линий магнитного поля, это означает, что рядом с проводом, по которому течет ток, должно быть магнитное поле.

Магнитное поле, создаваемое электрическим током, всегда ориентированы перпендикулярно направлению тока. Ниже приведен эскиз того, как выглядит магнитное поле вокруг провода, когда по нему течет ток. Мы используем \ (\ vec {B} \) для обозначения магнитного поля и стрелки на силовых линиях, чтобы показать направление магнитного поля. Обратите внимание на , что если нет тока, не будет магнитного поля.

Направление тока в проводе (проводе) показано центральной стрелкой. Кружки являются линиями поля, и они также имеют направление, указанное стрелками на линиях. Как и в случае с силовыми линиями электрического поля, чем больше количество линий (или чем они ближе друг к другу) в области, тем сильнее магнитное поле.

Важно: все наши обсуждения направлений полевых работ предполагают, что мы имеем дело с условным током .

Чтобы визуализировать эту ситуацию, поставьте ручку или карандаш прямо на стол. Круги центрируются вокруг карандаша или ручки и должны быть нарисованы параллельно поверхности стола. Кончик ручки или карандаша должен указывать в направлении тока.

Вы можете посмотреть на карандаш или ручку сверху, и карандаш или ручка будет точкой в ​​центре кругов. Направление силовых линий магнитного поля в этой ситуации — против часовой стрелки.

Чтобы было легче увидеть, что происходит, мы нарисуем только один набор круговых линий полей, но обратите внимание, что это только для иллюстрации.

Если вы положите лист бумаги за карандаш и посмотрите на него сбоку, то увидите, что круговые линии поля расположены сбоку, и трудно понять, что они круглые. Они проходят через бумагу. Помните, что линии поля имеют направление, поэтому, когда вы смотрите на лист бумаги сбоку, это означает, что круги входят в бумагу с одной стороны карандаша и выходят из бумаги с другой стороны.

Когда рисуем направления магнитных полей и токов, используем символы \ (\ odot \) и \ (\ otimes \).Символ \ (\ odot \) представляет собой стрелка, выходящая со страницы, и символ \ (\ время \) представляет собой стрелку, ведущую на страницу.

Значения символов легко запомнить, если вы подумаете о стрела с острым концом на голове и хвост с перьями в форме креста.

Датский физик Ганс Кристиан Эрстед однажды в 1820 году читал лекцию о возможности связи электричества и магнетизма друг с другом и в процессе убедительно продемонстрировал это с помощью эксперимента перед всем своим классом.Пропуская электрический ток через металлический провод, подвешенный над магнитным компасом, Эрстед смог вызвать определенное движение стрелки компаса в ответ на ток. То, что начиналось как предположение в начале занятия, в конце подтвердилось как факт. Излишне говорить, что Эрстеду пришлось пересматривать свои конспекты лекций для будущих уроков. Его открытие открыло дорогу совершенно новой отрасли науки — электромагнетизму.

Теперь мы рассмотрим три примера токоведущих проводов.Для каждого примера мы определим магнитное поле и проведем силовые линии магнитного поля вокруг проводника.

Магнитное поле вокруг прямого провода (ESBPT)

Направление магнитного поля вокруг токоведущей проводник показан на рисунке 10.1.

Рисунок 10.1: Магнитное поле вокруг проводника, когда вы смотрите на проводник с одного конца. (а) Ток течет со страницы и магнитное поле против часовой стрелки. (б) Ток течет в страницы, а магнитное поле — по часовой стрелке.Рисунок 10.2: Магнитные поля вокруг проводника, смотрящего на проводник сверху вниз. (а) Ток течет по часовой стрелке. (б) ток течет против часовой стрелки.

Направление магнитного поля

Используя направления, указанные на рисунках 10.1 и 10.2, попытайтесь найти правило, которое легко подскажет вам направление магнитного поля.

Подсказка: используйте пальцы. Возьмите проволоку в руки и попытайтесь найти связь между направлением большого пальца и направлением их сгибания.

Существует простой метод определения взаимосвязи между направлением тока, протекающего в проводнике, и направлением магнитного поля вокруг того же проводника. Метод называется Правило правой руки . Проще говоря, Правило правой руки гласит, что силовые линии магнитного поля, создаваемые токоведущим проводом, будут ориентированы в том же направлении, что и согнутые пальцы правой руки человека (в положении «автостоп»), при этом большой палец должен указывать внутрь. направление тока.

Ваша правая и левая рука уникальны в том смысле, что вы не можете повернуть одну из них, чтобы она находилась в том же положении, что и другая. Это означает, что важна правая часть правила. Вы всегда получите неправильный ответ, если воспользуетесь не той рукой.

Правило правой руки

Используйте Правило правой руки, чтобы нарисовать направления магнитных полей для следующих проводников с токами, текущими в направлениях, показанных стрелками. Первая задача была выполнена за вас.

1.

2.

3.

4.

5.

9002 6.

9002 6.

7.

8.

9.

10.

11.

12.

Магнитное поле вокруг проводника с током

Аппарат

  1. один \ (\ text {9} \) \ (\ text {V} \) аккумулятор с держателем

  2. два соединительных провода с зажимами типа «крокодил»

  3. компас

  4. секундомер

Метод

  1. Подключите провода к батарее, оставив один конец каждого провода неподключенным, чтобы цепь не замкнулась.

  2. Обязательно ограничивайте ток до \ (\ text {10} \) \ (\ text {seconds} \) за раз (вы можете спросить, у провода очень маленькое сопротивление, поэтому батарея разряжается квартира очень быстро). Это сделано для продления срока службы батареи, а также для предотвращения перегрева проводов и контактов батареи.

  3. Поднесите компас к проводу.

  4. Замкните цепь и посмотрите, что происходит с компасом.

  5. Поменяйте полярность батареи и замкните цепь. Понаблюдайте, что происходит с компасом.

Выводы

Используйте свои наблюдения, чтобы ответить на следующие вопросы:

  1. Создает ли ток, протекающий по проводу, магнитное поле?

  2. Присутствует ли магнитное поле, когда ток не течет?

  3. Зависит ли направление магнитного поля, создаваемого током в проводе, от направления тока?

  4. Как направление тока влияет на магнитное поле?

Магнитное поле вокруг токоведущей петли (ESBPV)

До сих пор мы рассматривали только прямые провода, по которым проходит ток, и магнитные поля вокруг них.Мы собираемся изучить магнитное поле, создаваемое кольцевыми витками провода, по которому проходит ток, потому что это поле имеет очень полезные свойства. Например, вы увидите, что мы можем создать однородное магнитное поле.

Магнитное поле вокруг петли проводника

Представьте себе две петли из проволоки, по которым течет ток (в противоположных направлениях) и которые параллельны странице вашей книги. Используя Правило правой руки, нарисуйте то, что, по вашему мнению, будет выглядеть магнитное поле в разных точках вокруг каждой из двух петель.В петле 1 ток течет против часовой стрелки, а в петле 2 ток течет по часовой стрелке.

Если вы сделаете петлю из проводника с током, то направление магнитного поля определяется применением правила правой руки к различным точкам петли.

Обратите внимание, что существует разновидность правила правой руки. Если вы заставите пальцы правой руки следовать направлению тока в петле, ваш большой палец будет указывать в том направлении, где выходят силовые линии.Это похоже на северный полюс (где силовые линии выходят из стержневого магнита) и показывает, какая сторона петли будет притягивать северный полюс стержневого магнита.

Магнитное поле вокруг соленоида (ESBPW)

Если мы теперь добавим еще одну петлю с током в том же направлении, то магнитное поле вокруг каждой петли можно будет сложить вместе, чтобы создать более сильное магнитное поле. Катушка из множества таких петель называется соленоидом . Соленоид — это цилиндрическая катушка с проволокой, действующая как магнит, когда электрический ток течет по проволоке.Картина магнитного поля вокруг соленоида аналогична картине магнитного поля вокруг стержневого магнита, который вы изучали в 10-м классе, у которого были определенные северный и южный полюсы, как показано на рисунке 10.3.

Рисунок 10.3: Магнитное поле вокруг соленоида.

Реальные приложения (ESBPX)

Электромагниты

Электромагнит представляет собой кусок провода, предназначенный для создания магнитного поля при прохождении через него электрического тока. Хотя все проводники с током создают магнитные поля, электромагнит обычно сконструирован таким образом, чтобы максимизировать силу магнитного поля, которое он создает для специальной цели.Электромагниты обычно используются в исследованиях, промышленности, медицине и потребительских товарах. Примером обычно используемого электромагнита являются защитные двери, например на двери магазина, которые открываются автоматически.

Как электрически управляемый магнит, электромагниты являются частью широкого спектра «электромеханических» устройств: машин, которые создают механическую силу или движение за счет электроэнергии. Возможно, наиболее очевидным примером такой машины является электродвигатель , который будет подробно описан в классе 12.Другими примерами использования электромагнитов являются электрические звонки, реле, громкоговорители и краны для свалок.

Электромагниты

Прицел

Магнитное поле создается, когда электрический ток течет по проводу. Одиночный провод не создает сильного магнитного поля, в отличие от провода, намотанного на железный сердечник. Мы исследуем это поведение.

Аппарат

  1. аккумулятор и держатель

  2. длина провода

  3. компас

  4. несколько гвоздей

Метод

  1. Если вы не проводили предыдущий эксперимент в этой главе, сделайте это сейчас.

  2. Согните провод в несколько катушек перед тем, как прикрепить его к батарее. Посмотрите, что происходит с отклонением стрелки компаса. Прогиб компаса стал сильнее?

  3. Повторите эксперимент, изменив количество и размер витков в проводе. Посмотрите, что происходит с отклонением по компасу.

  4. Намотайте провод на железный гвоздь, а затем прикрепите катушку к батарее.Посмотрите, что происходит с отклонением стрелки компаса.

Выводы

  1. Влияет ли количество катушек на силу магнитного поля?

  2. Железный гвоздь увеличивает или уменьшает силу магнитного поля?

Воздушные линии электропередачи и окружающая среда

Физическое воздействие

Линии электропередач — обычное явление для всей нашей страны.Эти линии подводят электричество от электростанций к нашим домам и офисам. Но эти линии электропередач могут иметь негативное воздействие на окружающую среду. Одна из опасностей, которые они представляют, — это летающие на них птицы. Защитник природы Джессика Шоу провела последние несколько лет, изучая эту угрозу. Фактически, линии электропередач представляют собой основную угрозу для синего журавля, национальной птицы Южной Африки, в Кару.

«Нам повезло, что в Южной Африке обитает широкий спектр видов птиц, в том числе много крупных птиц, таких как журавли, аисты и дрофы.К сожалению, существует множество линий электропередач, которые могут воздействовать на птиц двояко. Они могут быть поражены электрическим током, когда садятся на некоторые типы пилонов, а также могут быть убиты, столкнувшись с линией, если они влетят в нее, либо от удара о веревку, либо после удара о землю. Эти столкновения часто случаются с крупными птицами, которые слишком тяжелы, чтобы избежать линии электропередачи, если они видят ее только в последнюю минуту. Другие причины, по которым птицы могут столкнуться, включают плохую погоду, полет стаями и отсутствие опыта у молодых птиц.

В течение последних нескольких лет мы изучали серьезное влияние столкновений линий электропередач на «Голубых журавлей» и «Дроф Людвига». Это два наших эндемичных вида, а это значит, что они встречаются только в южной части Африки. Это большие птицы, которые живут долго и размножаются медленно, поэтому популяции могут не восстановиться после высокой смертности. Мы прошли и проехали под линиями электропередач через Оверберг и Кару, чтобы подсчитать мертвых птиц. Данные показывают, что ежегодно тысячи этих птиц гибнут в результате столкновений, а дрофа Людвига в настоящее время занесена в список исчезающих видов из-за такого высокого уровня неестественной смертности.Мы также ищем способы уменьшить эту проблему и работаем с Eskom над тестированием различных устройств для маркировки линий. Когда на линиях электропередач вешают маркеры, птицы могут видеть линию электропередач с большого расстояния, что дает им достаточно времени, чтобы избежать столкновения ».

Воздействие полей

Тот факт, что вокруг линий электропередачи создается поле, означает, что они потенциально могут иметь воздействие на расстоянии. Это было изучено и продолжает оставаться предметом серьезных дискуссий.На момент написания руководства Всемирной организации здравоохранения по воздействию на человека электрических и магнитных полей указано, что нет четкой связи между воздействием магнитных и электрических полей, с которыми население сталкивается от линий электропередач, поскольку это поля чрезвычайно низкой частоты. .

Шум от линии электропередачи может мешать радиосвязи и радиовещанию. По сути, линии электропередач или связанное с ними оборудование неправильно генерируют нежелательные радиосигналы, которые отменяют или конкурируют с полезными радиосигналами.Шум от линии электропередачи может повлиять на качество приема радио и телевидения. Также может произойти нарушение радиосвязи, например, любительского радио. Потеря критически важной связи, такой как полиция, пожарная охрана, военные и другие подобные пользователи радиочастотного спектра, может привести к еще более серьезным последствиям.

Групповое обсуждение:

Когда молния поражает корабль или самолет, она может повредить или иным образом изменить его магнитный компас. Были зарегистрированы случаи, когда при ударе молнии полярность компаса менялась, так что стрелка указывала на юг, а не на север.

Зарегистрируйтесь, чтобы получить стипендию и возможности карьерного роста. Используйте практику Сиявулы, чтобы получить наилучшие возможные оценки.

Зарегистрируйтесь, чтобы разблокировать свое будущее

Магнитные поля

Упражнение 10.1

Привести доказательства существования магнитного поля возле токоведущего провода.

Если вы поднесете компас к проводу, по которому течет ток, стрелка на компасе отклонится. Поскольку компасы работают, указывая вдоль силовых линий магнитного поля, это означает, что вблизи провода, по которому течет ток, должно быть магнитное поле.Если ток перестанет течь, компас вернется в исходное направление. Если ток снова начнет течь, отклонение произойдет снова.

Опишите, как вы могли бы использовать правую руку, чтобы определять направление магнитного поля вокруг проводника с током.

Мы используем правило правой руки, которое гласит, что силовые линии магнитного поля, создаваемые токоведущим проводом, будут ориентированы в том же направлении, что и согнутые пальцы правой руки человека (в положении «автостоп»), при этом большой палец указывает по направлению тока:

Со страницы

на страницу

Используйте Правило правой руки, чтобы найти направление магнитных полей в каждой из точек, обозначенных A — H на следующих диаграммах.

Добавить комментарий

Ваш адрес email не будет опубликован. Обязательные поля помечены *